You are on page 1of 75

_

DO NOT OPEN THIS BOOKLET UNTIL YOU ARE ASKED TO DO SO

VAJIRAM & RAVI


Prelims Test Series – 2023
Full Length Test – 01 – (R4346)

General Studies
Full Test-1

Full Syllabus Cover

2023
_
1. Consider the following statements: 4. With reference to the ‘G20 Summit’, consider
1. Unlike real GDP, nominal GDP does not the following statements:
account for any inflation in the economy. 1. Sherpa Track coordinates and prepares
2. During inflationary periods, nominal GDP the agenda for the annual G20 summit
will be higher than real GDP. meetings.
Which of the statements given above is/are 2. Finance Track is the meeting of the
correct? Finance Ministers and the Central Bank
(a) 1 only Governors of the member countries.
(b) 2 only Which of the statements given above is/are
(c) Both 1 and 2
correct?
(d) Neither 1 nor 2
(a) 1 only
(b) 2 only
2. With reference to the Indian economy,
(c) Both 1 and 2
consider the following statements:
(d) Neither 1 nor 2
1. A decline in Current Account Deficit
would lead to an appreciation of Indian
currency. 5. Consider the following statements with
2. An increase in the Reverse Repo Rate reference to the Sovereign Green Bond:
leads to an increase in the borrowing 1. It is issued by the Reserve Bank of India.
cost of loans. 2. It is a part of the Government's overall
3. Increase in interest rate by the Federal market borrowing.
Reserve of the United States of America 3. It is a rupee-denominated paper that will
would lead to an increase in capital not be available to foreign investors.
inflow in the Indian economy. Which of the statements given above is/are
Which of the statements given above are correct?
correct? (a) 1 and 2 only
(a) 1 and 2 only (b) 2 only
(b) 2 and 3 only (c) 1 and 3 only
(c) 1 and 3 only (d) 1, 2 and 3
(d) 1, 2 and 3
6. With reference to the ‘Online Gaming
3. Consider the following statements: Industry in India’, consider the following
1. Purchase of government bonds from
statements:
banks by the Reserve Bank of India
1. Foreign Direct Investment (FDI) is not
(RBI) most likely leads to inflation.
allowed under the automatic route in the
2. The sale of dollars in the market by the
industry.
RBI most likely leads to rupee
2. Goods and Services Tax (GST) is
appreciation.
3. During inflation, the RBI is likely to applicable to all online games.
increase the benchmark interest rates 3. Ministry of Electronics and Information
including the repo rate. Technology (MeitY) acts as the nodal
Select the correct answer using the code agency for online gaming in India.
given below: Which of the statements given above are
(a) 1 only correct?
(b) 1 and 2 only (a) 1 and 2 only
(c) 2 and 3 only (b) 2 and 3 only
(d) 1, 2 and 3 (c) 1 and 2 only
(d) 1, 2 and 3

1
Vajiram & Ravi Prelims Test Series (2023)
Full Length Test No. – 01 (R4346)
_
7. Which of the following items are exempted (c) Both 1 and 2
from the purview of the Goods and Services (d) Neither 1 nor 2
Tax in India?
1. Sale of land 11. Which of the following provisions under the
2. All passenger air travel to the north- Constitution of India ensures the “Principle of
eastern states Checks and Balances” in Indian Polity?
3. Pre-packaged and labelled milk 1. Parliament is prohibited from
4. Distribution of electricity discussing the judicial conduct of a
Select the correct answer using the code Supreme Court judge
given below: 2. Appointment of the High Court judges
(a) 1, 2 and 3 only by the Executive Head of the State
(b) 2 and 3 only
3. Judicial review exercised by the
(c) 1 and 4 only
judiciary over legislative actions
(d) 1, 2, 3 and 4
Select the correct answer using the code
8. A downward-sloping yield curve in an given below:
economy indicates: (a) 1 and 2 only
(a) economic slowdown in the future (b) 2 and 3 only
(b) imminent rise in interest rates (c) 1 and 3 only
(c) stable economy with no change in (d) 1, 2 and 3
inflation
(d) steep rupee depreciation 12. Consider the following statements:
1. The Supreme Court has clearly defined
9. With reference to ‘Open Offer’, consider the the basic structure of the Constitution in
following statements: the Kesavananda Bharati case.
1. It is a formal offer made by a company to 2. Limited power of Parliament to amend
the shareholders of a target company to the Constitution is a part of the basic
sell their shares at a specified price. structure of the Constitution.
2. It can be initiated only if an acquirer Which of the statements given above is/are
holds the majority of the public correct?
shareholding in the target company. (a) 1 only
3. The share price offered in the open offer (b) 2 only
cannot be less than the market price of
(c) Both 1 and 2
the shares at the time of acquisition.
(d) Neither 1 nor 2
Which of the statements given above is/are
correct?
(a) 1 only 13. With reference to ‘Money Bill’, consider the
(b) 1 and 2 only following statements:
(c) 3 only 1. It can be introduced only in the Lok
(d) 2 and 3 only Sabha and not in the Rajya Sabha.
2. A joint sitting of both Houses of
10. With reference to Surety Bond, consider the Parliament can be convened in case
following statements: there is a deadlock.
1. It can be used as a substitute for bank 3. When it is presented to the President of
guarantees in government procurement India, it is obligatory for the President of
tenders. India to give his/her assent.
2. It is provided by the insurance company Which of the statements given above is/are
on behalf of the contractor to the entity correct?
which is awarding the project. (a) 1 only
Which of the statements given above are (b) 1 and 2 only
correct? (c) 3 only
(a) 1 only (d) 2 and 3 only
(b) 2 only
2
Vajiram & Ravi Prelims Test Series (2023)
Full Length Test No. – 01 (R4346)
_
14. Which of the following is not correct with 17. With reference to the ‘Delimitation
respect to the Right to Freedom of Speech Commission’, consider the following
and Expression under Article 19 of the statements:
Constitution of India? 1. The Minister of Home Affairs is the ex-
(a) It is enforceable against private officio member of the Delimitation
individuals and entities. Commission.
(b) It is available to both citizens and 2. It identifies seats to be reserved for
foreigners except enemy aliens. Scheduled Castes and Scheduled Tribes
(c) It cannot be restricted on any condition in Parliamentary and State Assembly
elections.
except those mentioned under Article 19.
3. The Parliament can modify the orders of
(d) It includes the right to receive and impart
the Delimitation Commission only by a
information through social media.
special majority.
Which of the statements given above is/are
15. In India, Directorate of Enforcement (ED) is a correct?
multi-dimensional organisation investigating (a) 1 only
economic offences under which of the (b) 2 only
following legislations? (c) 2 and 3 only
1. Prevention of Money Laundering Act, (d) 1 and 3 only
2002
2. Foreign Exchange Management Act, 18. Consider the following statements:
1999 1. The President of India can remove the
3. Prevention of Corruption Act, 1988 Governor of a State from office without
4. Fugitive Economic Offenders Act, 2018 assigning any reason.
Select the correct answer using the code 2. 2.The Governor of a State may be
given below: dismissed from office if the Union
(a) 1 and 2 only Government loses confidence in him/her.
(b) 3 and 4 only 3. Sarkaria Commission had recommended
(c) 1, 2 and 4 only incorporating a provision to impeach the
(d) 1, 2, 3 and 4 Governor by the State Legislative
Assembly.
16. Consider the following statements with Which of the statements given above is/are
correct?
respect to the Memorandum of Procedure
(a) 1 only
(MoP) for the appointment of the Chief
(b) 2 and 3 only
Justice of India (CJI):
(c) 3 only
1. The Ministry of Law and Justice (d) 1 and 3 only
seeks the recommendation of the
outgoing CJI for the appointment of 19. Which one of the following statements best
the next CJI. describes the term 'equality' in the context of
2. The next CJI must be appointed two Indian Polity?
months before the retirement of the (a) Equal treatment of all regardless of
outgoing CJI. individual characteristics and differences
Which of the statements given above (b) Absence of special privileges in favour of
is/are correct? any individual
(a) 1 only (c) Ensuring the equal distribution of income
(b) 2 only and wealth in society
(c) Both 1 and 2 (d) Promotion of unity and integrity among
(d) Neither 1 nor 2 all sections of society

3
Vajiram & Ravi Prelims Test Series (2023)
Full Length Test No. – 01 (R4346)
_
20. With reference to Indian Polity, the 'Doctrine Select the correct answer using the code
of Proportionality' is applied by the Judiciary given below:
to test: (a) 1, 2 and 4 only
(a) the reasonable balance between laws (b) 1, 4 and 5 only
and government actions (c) 2, 3 and 5 only
(b) validity of a Constitutional Amendment (d) 1, 2, 3, 4 and 5
Act
(c) encroachment by the Central 24. The drainage basin of the Brahmaputra
Government in the legislative jurisdiction River covers which of the following states?
of a State 1. Assam
(d) validity of the Speaker’s decision on the 2. Tripura
anti-defection laws 3. Nagaland
4. Sikkim
21. Consider the following statements with 5. Meghalaya
reference to Millets: Select the correct answer using the code
1. The earliest evidence of millets in India is given below:
found in the Indus Valley Civilization. (a) 1, 2 and 4 only
2. They are annual grasses grown in dry (b) 2, 3 and 5 only
areas of temperate and subtropical (c) 1, 3, 4 and 5 only
regions. (d) 1, 2, 3, 4 and 5
3. They are naturally gluten free.
25. Consider the following pairs:
4. They don't need a specific photoperiod to
flower. Sl. Lake Location
Which of the statements given above are No.
correct?
(a) 1and 2 only 1. Bhimtal : Himachal
(b) 2 and 3 only Pradesh
(c) 1, 3 and 4 only
2. Rewalsar : Uttarakhand
(d) 1, 2, 3 and 4
3. Tso Kar : Ladakh
22. With reference to Regenerative Agriculture,
consider the following statements: 4. Gurudongmar : Sikkim
1. It focuses on improving the health of the
Which of the pairs given above is/are
soil.
correctly matched?
2. It adopts the mono-cropping method to
(a) 1 and 2 only
increase productivity.
3. It aims to reduce the use of synthetic (b) 3 and 4 only
inputs such as herbicides and pesticides. (c) 2 and 3 only
Which of the statements given above is/are (d) 1 and 4 only
correct?
(a) 1 only 26. Consider the following countries:
(b) 2 only 1. Belarus
(c) 1 and 3 only 2. Estonia
(d) 2 and 3 only 3. Hungary
4. Czech Republic
23. Which of the following are ideal stargazing 5. Poland
spots in India? Which of the above have borders with
1. Katao in Sikkim Ukraine?
2. Churu in Rajasthan (a) 1, 2 and 4 only
3. Burnpur in West Bengal (b) 3, 4 and 5 only
4. Spiti Valley in Himachal Pradesh (c) 1, 3 and 5 only
5. Kovalam in Kerala (d) 1, 2, 3, 4 and 5
4
Vajiram & Ravi Prelims Test Series (2023)
Full Length Test No. – 01 (R4346)
_
27. With reference to the Western Disturbances, 30. Consider the following pairs:
consider the following statements:
1. These are low-pressure storms that SI. Local Description
originate from subtropical high-pressure No. Wind
1. Simoon : Cold wind that blows
regions.
towards the eastern
2. Jet streams blowing south of the shore of the Adriatic
Himalayas bring in western disturbances Sea
in India. 2. Sirocco : Warm and humid
3. They are beneficial for the wheat and wind that causes
apple crops in the country. dusty conditions
Which of the statements given above is/are along the north coast
of Africa
correct?
3. Bora : Intense hot wind in
(a) 1 only
the Arabian desert
(b) 1 and 2 only 4. Pampero : Cold wind that blows
(c) 3 only over Argentina
(d) 1, 2 and 3 Which of the pairs given above are correctly
matched?
28. With reference to Steppe climatic region, (a) 1 and 2 only
consider the following statements: (b) 1 and 3 only
(c) 2 and 4 only
1. It is a dry grassy plain lying between the
(d) 3 and 4 only
equator and the tropical region.
2. Chinook, a hot local wind, blows through 31. In India, which of the following are
the region. considered “Digital Public Goods”?
3. It is ideal for extensive mechanised 1. Unified Payment Interface
wheat cultivation. 2. Aarogya Setu application
Which of the statements given above are 3. Core Banking Solutions
4. Open Network for Digital Commerce
correct?
5. Aadhaar Identification system
(a) 1 and 2 only Select the correct answer using the code
(b) 2 and 3 only given below:
(c) 1 and 3 only (a) 1, 2 and 3 only
(d) 1, 2 and 3 (b) 1, 2, 4 and 5 only
(c) 3, 4 and 5 only
29. The perceived centre of the Sun’s disk is in (d) 1, 2, 3, 4 and 5
the same plane as the Equator when:
32. Which of the following can be treated by
(a) subsolar point hits the Tropic of Cancer using "Gene Therapy"?
around June 21 1. Inherited blindness
(b) subsolar point hits the Tropic of 2. Haemophilia
Capricorn around December 21 3. Fat metabolism disorder
(c) the Earth's axis is not tilting toward or 4. Parkinson's disease
away from the sun. Select the correct answer using the code
given below:
(d) the Earth is closest to the Sun
(a) 2 and 4 only
(b) 1, 3 and 4 only
(c) 2 and 3 only
(d) 1, 2, 3 and 4

5
Vajiram & Ravi Prelims Test Series (2023)
Full Length Test No. – 01 (R4346)
_
33. With reference to the India Biological Data 37. Which of the following are the benefits of
Bank, consider the following statements: food irradiation technology?
1. It is mandated to archive all life science 1. Improving the shelf life of foods
data generated from publicly funded 2. Elimination of harmful insects
research in India. 3. Assist in early sprouting
2. It stores the genomic sequences of 4. Sterilisation of foods
crops, humans and animals in the Select the correct answer using the code
database. given below:
Which of the statements given above is/are (a) 1 and 3 only
correct? (b) 1, 2 and 4 only
(a) 1 only (c) 2, 3 and 4 only
(b) 2 only (d) 1, 2, 3 and 4
(c) Both 1 and 2
(d) Neither 1 nor 2 38. Consider the following statements with
reference to ‘Mock Meat’:
34. The terms ‘Cry1Ac’ and ‘Cry2Ab, sometimes 1. It is a plant-based product made from
seen in the media, are related to: vegetarian ingredients.
(a) genetically modified crops 2. Its texture and consistency are the same
(b) environmental allergens as that of conventional meat.
(c) cryogenic technologies 3. It has high fibre and low-fat content.
(d) digital cloud services Which of the statements given above are
correct?
35. Which one of the following statements is not (a) 1 and 2 only
correct with reference to ‘Exoplanets’? (b) 2 and 3 only
(a) They are planets that orbit stars outside (c) 1 and 3 only
of our solar system. (d) 1, 2 and 3
(b) They can be observed through the direct
imaging method by telescope. 39. A species named ‘Halteria’ was in the news
(c) All exoplanets have a smaller mass than recently in the context of which one of the
Earth. following?
(d) They are made up of elements similar to (a) Cure for Alzheimer's disease
those found in Earth. (b) Sustainable textile fibre
(c) Organism that feeds on viruses
36. Which of the following are the benefits of (d) Source of biodiesel
nano fertilisers?
1. Control the viral and fungal diseases in 40. Which of the following are artificial fruit
plants ripeners?
2. Prevention of soil leaching 1. Lauryl alcohol
3. Promotion of precision farming 2. Ethephon
4. No side effects on human health 3. Acetylene gas
Select the correct answer using the code 4. Ethylene
given below: Select the correct answer using the code
(a) 1 and 3 only given below:
(a) 1 and 4 only
(b) 2, 3 and 4 only
(b) 2, 3 and 4 only
(c) 1, 2 and 4 only
(c) 1, 2 and 3 only
(d) 1, 2, 3 and 4
(d) 1, 2, 3 and 4

6
Vajiram & Ravi Prelims Test Series (2023)
Full Length Test No. – 01 (R4346)
_
41. The term ‘COWCLIP 2.0’, mentioned in the 45. ‘Naegleria fowleri’, recently seen in the
news, is a: news, is a:
(a) working group for implementation of (a) single-celled organism that can infect
goals under the Paris Agreement humans
(b) committee to discuss the impacts of (b) variety of neelakurinji flower found in the
genetic engineering in animals Himalayas
(c) research project that provides an (c) invasive plant species threatening the
assessment of ocean waves and coastal biodiversity of Western Ghats
hazards (d) soil-binding plant which checks siltation
(d) agency that finances Artificial Intelligence
in rivers
based research
46. Recently, Sodium tetrafluoroborate (NaBF4)
42. With reference to Green Hydrogen, consider
was found to be useful for industries
the following statements:
1. It is produced by the process of producing green energy. In this context,
electrolysis. which of the following are the correct
2. It can be converted into synthetic gas for applications of Sodium tetrafluoroborate?
mobility purposes. 1. Neutralizing agent in the textile industry
Which of the statements given above is/are 2. Lubricant in the automotive industry
correct? 3. Flux in the glass industry
(a) 1 only 4. Colouring substance in the food industry
(b) 2 only Select the correct answer using the code
(c) Both 1 and 2 given below:
(d) Neither 1 nor 2 (a) 1 and 3 only
(b) 2 and 4 only
43. The ‘AWARe Action Plan’ launched at the (c) 1, 2 and 4 only
27th Conference of Parties (COP27) of the (d) 1, 2, 3 and 4
UNFCCC aims to:
(a) accelerate the deployment of offshore 47. Which of the following factors are
wind power to tackle the energy crisis responsible for the occurrence of debris
(b) build consensus on a multilateral treaty avalanches and landslides in the Western
for resource exploration in the Arctic Ghats?
(c) foster political efforts in establishing pan-
1. Steep slopes with vertical cliffs
African water hubs
2. Tectonically active zone
(d) focus on high-potential energy sectors in
3. High temperature changes
developing countries
4. Heavy rainfall over a short period
44. Consider the following statements with Select the correct answer using the code
reference to Black Carbon Aerosol: given below:
1. It forms due to the incomplete (a) 1, 3 and 4 only
combustion of biofuels. (b) 2 and 3 only
2. It has a strong light absorption capacity (c) 1, 2 and 4 only
that results in the warming of the earth. (d) 1, 2, 3 and 4
Which of the statements given above is/are
correct? 48. Which one of the following is not a medicinal
(a) 1 only plant found in the Himalayas?
(b) 2 only (a) Salampanja
(c) Both 1 and 2 (b) Mahameda
(d) Neither 1 nor 2 (c) Arundinella ciliata
(d) Kshirkakoli

7
Vajiram & Ravi Prelims Test Series (2023)
Full Length Test No. – 01 (R4346)
_
49. It is a biodiversity-rich site having historical 52. In medieval India, the term ‘Amir-i-
significance which is a home to the Indian Chahalgani’ referred to:
Pangolin and Slender Loris. The site (a) Foreign Invaders
features various megalithic structures, Tamil (b) Trusted Nobles
Brahmi Inscriptions, Jain Beds and 2200- (c) Women Spies
year-old rock-cut temples. It also has the (d) Agricultural labourers
Anaikondan lake built during the reign of the
Pandyas in the 16th century. It is also Tamil 53. Dr. B.R Ambedkar was actively associated
Nadu’s first biodiversity Heritage site. with which of the following?
This is the most likely description of: 1. All India Harijan Sevak Sangh
(a) Borjuli Wild Rice Site 2. All India Scheduled Castes Federation
(b) Nallur Tamarind Grove 3. Independent Labour Party
(c) Arittapatti Select the correct answer using the code
(d) Chabimura given below:
(a) 1 only
50. Which of the following Biosphere Reserves (b) 2 and 3 only
in India are part of the Man and the (c) 1 and 3 only
Biosphere Programme (MAB) programme of (d) 1, 2 and 3
UNESCO?
1. Gulf of Mannar 54. Consider the following statements:
2. Dibru-Saikhowa The Mountbatten Plan provided for:
3. Nokrek 1. two Dominions and two Constituent
4. Pachmarhi Assemblies
5. Kachchh 2. referendum in the North-West Frontier
Select the correct answer using the code Province
given below: 3. independence of princely states
(a) 1, 3 and 4 only Which of the statements given above is/are
(b) 1, 2, 4 and 5 only correct?
(c) 2, 3 and 5 only (a) 1 and 3 only
(d) 1, 2, 3, 4 and 5 (b) 1 and 2 only
(c) 3 only
51. In this context of colonial India, the (d) 2 and 3 only
Government of India Act of 1935
recommended the establishment of: 55. With reference to Indian history, consider the
1. Council of India following sites:
2. Bicameral legislature in provinces 1. Sammed Shikharji
3. Federal form of government 2. Shatrunjaya
4. Central Public Service Commission 3. Thalaivetti Muniyappan
Select the correct answer using the code 4. Ashtapad
given below: Which of the above are Jain pilgrimage
(a) 1 and 2 only sites?
(b) 3 and 4 only (a) 1 and 2 only
(c) 2 and 3 only (b) 1, 2 and 4 only
(d) 1 and 4 only (c) 3 and 4 only
(d) 2, 3 and 4 only

8
Vajiram & Ravi Prelims Test Series (2023)
Full Length Test No. – 01 (R4346)
_
56. With reference to ‘Medieval India’, consider 59. With reference to the ‘Anti-Compromise
the following pairs: Conference’, consider the following
statements:
Sl. Foreign Visited during 1. It was convened by Muhammad Ali
No. Traveller the rule of Jinnah at the Lahore session of the
Muslim League in 1940.
1. Abdur Razzak : Pala 2. It was aimed at safeguarding the political
rights of Muslims in India.
Which of the statements given above is/are
2. Ibn Battuta : Tughlaq
correct?
(a) 1 only
3. Marco Polo : Pandya (b) 2 only
(c) Both 1 and 2
(d) Neither 1 nor 2
4. Niccolo Conti : Khalji
60. Who among the following established the
'Mahila Seva Mandal' and 'Native Female
Which of the pairs given above are correctly School' in Pune?
matched? (a) Aruna Asaf Ali
(a) 1 and 4 only (b) Jhalkari Bai
(b) 2 and 3 only (c) Savitribai Phule
(c) 1, 3 and 4 only (d) Sarla Devi Chaudharani
(d) 2, 3 and 4 only
61. Consider the following statements:
57. With reference to ‘Indian History’, consider 1. If the interest rate prevailing in the
the following statements: economy is high, the foreign exchange
1. The first Turkish invasion of India rate is likely to decline.
happened during the reign of Iltutmish. 2. If the country is experiencing a surplus
2. Mahmud of Ghazni is known for his raid trade balance, the domestic currency is
on Somnath Temple during his invasion likely to appreciate in its value.
of India. Which of the statements given above is/are
3. In the Battle of Tarain, Prithviraj correct?
(a) 1 only
Chauhan defeated Ghurid forces and
(b) 2 only
later captured Tabarhinda region.
(c) Both 1 and 2
Which of the statements given above is/are
(d) Neither 1 nor 2
correct?
(a) 1 only 62. The Government of India announces
(b) 2 only Minimum Support Price (MSP) for which of
(c) 2 and 3 only the following crops?
(d) 1 and 3 only 1. Jowar
2. Mustard seed
58. With reference to the modern history of 3. Ginger
India, ‘Damin-i-Koh’ refers to: 4. Cotton
(a) duties imposed on the export of Indian 5. Jute
handicrafts 6. Sunflower
(b) revenue system imposed by the British 7. Coffee
authorities in North-east India Select the correct answer using the code
(c) advances given by the English East India given below:
Company to merchants in Bengal (a) 1, 3 and 4 only
(d) land exclusively demarcated for the (b) 1, 2, 4, 5 and 6 only
santhals to practise agriculture (c) 2, 3, 5 and 7 only
(d) 1, 2, 4, 5, 6 and 7 only
9
Vajiram & Ravi Prelims Test Series (2023)
Full Length Test No. – 01 (R4346)
_
63. Which of the following measures would 66. India is a member of which of the following
result in an increase in the money supply in groups that facilitate development and trade
the economy? in Semiconductor technologies?
1. Increase in the reserve requirement by
1. Trade and Technology Council
the Central Bank
2. Chip 4 alliance
2. Printing of new currency
3. Quantitative easing by the Central Bank 3. Quad Semiconductor Supply Chain
4. Deposit of currency in commercial banks Initiative
by the public 4. World Semiconductor Council
Select the correct answer using the code Select the correct answer using the code
given below: given below:
(a) 1 and 2 only
(a) 1 and 4 only
(b) 1, 2 and 3 only
(b) 2 and 3 only
(c) 1, 3 and 4 only
(d) 2, 3 and 4 only (c) 3 only
(d) 2, 3 and 4 only
64. Consider the following statements with
reference to National Company Law 67. Consider the following statements with
Appellate Tribunal (NCLAT): reference to the ‘Digital Agristack’:
1. It is a quasi-judicial body established
1. It is a centralised platform for farmers
under the Companies Act, 2013.
that gives them access to end-to-end
2. It hears appeals against any order
passed by the Competition Commission services in the food value chain.
of India (CCI). 2. Each farmer will have a unique digital
3. No court has jurisdiction in respect of any identification linked to their digital
decision passed by the tribunal. Aadhaar.
Which of the statements given above is/are 3. Private companies have been authorised
correct?
for developing the farmers’ database.
(a) 1 only
Which of the statements given above is/are
(b) 1 and 2 only
(c) 3 only correct?
(d) 2 and 3 only (a) 1 only
(b) 3 only
65. With reference to ‘Convertible Note’, (c) 1 and 2 only
consider the following statements: (d) 2 and 3 only
1. It is a financial instrument used by start-
ups to raise capital without the
68. In India, which one of the following regulates
requirement of a valuation report.
2. It can be converted into equity shares the ‘Spot’ and ‘Derivative’ markets?
within 5 years from the date of issuance. (a) Reserve Bank of India (RBI)
3. It is issued by Private Limited Companies (b) Commission for Agricultural Costs &
registered under the Startup India Prices (CACP)
scheme. (c) Securities and Exchange Board of India
Which of the statements given above is/are
(SEBI)
correct?
(d) National Commodity & Derivatives
(a) 1 only
(b) 1 and 2 only Exchange Limited (NCDEX)
(c) 3 only
(d) 1, 2 and 3

10
Vajiram & Ravi Prelims Test Series (2023)
Full Length Test No. – 01 (R4346)
_
69. It is a form of cloud computing that offers 72. Which of the following are the functions of
essential computing, storage, and the Department for Promotion of Industry
networking resources on demand. It enables and Internal Trade (DPIIT)?
consumers to reduce the maintenance of on- 1. It administers the Geographical
premises data centres, save money on Indications of Goods (Registration and
hardware costs, and gain real-time business Protection) Act of 1999 to protect
insights. Intellectual Property Rights.
The above paragraph best describes which 2. It is the nodal department for the
one of the following cloud computing formulation of the policy of the
services? Government on Foreign Direct
(a) Software as a Service Investment (FDI).
(b) Infrastructure as a Service 3. It undertakes the promotion of
(c) Platform as a Service productivity and quality in the industrial
(d) Desktop as a Service sector through the National Productivity
Council (NPC).
70. Consider the following pairs: Select the correct answer using the code
given below:
Sl. Harappan Known for (a) 1 and 2 only
No. Town (b) 2 and 3 only
1. Chanhudaro : City without a (c) 1 and 3 only
(d) 1, 2 and 3
walled citadel
2. Kalibangan : Seven fire
73. If a particular area is brought under the Fifth
altars Schedule of the Constitution of India, which
3. Lothal : Practice of one of the following will be the necessary
mixed cropping consequence of it?
4. Rakhigarhi : Terracotta (a) It can constitute Tribal Advisory Councils
manufacturing to advise on the welfare of the
Scheduled Tribes.
Which of the pairs given above are correctly (b) It can establish Autonomous District
matched? Councils (ADCs).
a) 1 and 2 only (c) It falls outside the executive authority of
b) 2 and 3 only the State concerned.
c) 3 and 4 only (d) The Governor of the State gets
d) 1, 2 and 4 only empowered to organise and re-organise
the Scheduled Area.
71. In India, who among the following is the final
authority in a dispute between the Central 74. In India, women’s right to make their own
Government and a State or between two reproductive decisions is protected under
States? which one of the following Articles of the
(a) Parliament Constitution of India?
(b) President (a) Article 15
(c) Central Government (b) Article 19
(d) Supreme Court (c) Article 21
(d) Article 23

11
Vajiram & Ravi Prelims Test Series (2023)
Full Length Test No. – 01 (R4346)
_
75. Which of the following institutions/authorities 78. With reference to the United Nations Country
are created under the Wildlife (Protection) Team (UNCT), consider the following
Act of 1972? statements:
1. National Board for Wildlife 1. It includes all UN entities working on
2. National Biodiversity Authority sustainable development in programme
3. Central Zoo Authority countries.
4. National Tiger Conservation Authority 2. It is led by the UN Resident Coordinator
Select the correct answer using the code in respective countries.
given below: 3. The UN Sustainable Development Group
(a) 1 and 2 only oversees the coordination of UNCT.
(b) 1, 3 and 4 only Which of the statements given above are
(c) 2, 3 and 4 only correct?
(d) 1, 2, 3 and 4 (a) 1 and 2 only
(b) 2 and 3 only
76. With reference to United Nations Convention (c) 1 and 3 only
on Biological Diversity (CBD), consider the (d) 1, 2 and 3
following statements:
1. It is a multilateral treaty that was opened 79. With reference to ‘University Grant
for signature at the Earth Summit in Rio Commission (UGC)’, consider the following
De Janeiro in 1992. statements:
2. It has adopted the Cartagena Protocol to 1. It was set up based on the
ensure the safe handling, transport and recommendations of the Kothari
use of Living Modified Organisms Commission in 1964.
(LMOs). 2. It provides approval to foreign
3. It is legally binding on the member universities to set up campuses in India.
countries. Which of the statements given above is/are
Which of the statements given above are correct?
correct? (a) 1 only
(a) 1 and 2 only (b) 2 only
(b) 2 and 3 only (c) Both 1 and 2
(c) 1 and 3 only (d) Neither 1 nor 2
(d) 1, 2 and 3
80. Which one of the following best describes
77. The ‘Moscow Format’, sometimes seen in the term ‘Carbon Leakage’?
the news, is a/an: (a) It is an illegal dumping of hazardous
(a) intergovernmental military alliance waste containing carbon in the
(b) diplomatic process to facilitate the environment.
national reconciliation process in (b) It is a process of storing leaked carbon
Afghanistan dioxide from thermal power plants.
(c) forum that engages in exploration of oil (c) The reduction of carbon emissions in one
and gas resources in the Arctic region sector leads to an equivalent carbon
(d) platform for a peaceful resolution to the emission increase in another sector.
Ukrainian-Russian conflict (d) It is a practice by companies to relocate
to countries with less stringent
regulations on carbon emissions.

12
Vajiram & Ravi Prelims Test Series (2023)
Full Length Test No. – 01 (R4346)
_
81. Arrange the following Himalayan landforms 84. Consider the following pairs:
in India from north to south with respect to
their occurrence of formation: Sl. Sacred Grove State
1. V-Shaped valleys No.
2. Glacial lakes
3. Estuaries 1. Kovil Kaadugal : Tamil Nadu
4. Meanders
2. Orans : Rajasthan
Select the correct answer using the code
given below: 3. Banis : Odisha
(a) 1-2-4-3
(b) 2-1-3-4 4. Sarana : Karnataka
(c) 2-4-1-3
(d) 2-1-4-3 Which of the pairs given above are correctly
matched?
82. Consider the following pairs: a) 1 and 2 only
b) 2, 3 and 4 only
Sl. Community Region
No. c) 3 and 4 only
d) 1, 2, 3 and 4
1. Kuki Chin : Bangladesh
2. Rohingya : Northern
Myanmar 85. With reference to ‘Sahitya Akademi Award’,
3. Hausa : Sudan - Ethiopia consider the following statements:
Border 1. It is given only to a book written in one of
the official languages under the Eighth
Schedule of the Constitution of India.
Which of the pairs given above are correctly
matched? 2. It is not given to a work of translation.
(a) 1 and 2 only 3. It cannot be given posthumously.
(b) 2 and 3 only Which of the statements given above is/are
(c) 1 and 3 only correct?
(d) 1, 2 and 3 (a) 1 only
(b) 2 only
83. Consider the following statements: (c) 2 and 3 only
Statement 1: Mass movements are more
(d) 1 and 3 only
active over weathered slopes than on un-
weathered materials.
86. Which of the following are in Nilgiri
Statement 2: Weathering is a necessary
Biosphere Reserve?
condition for mass movements.
(a) Mudhumali, Bandipur, Silent Valley,
Which one of the following is correct in
respect of the above statements? National Parks and Wayanad Wildlife
(a) Both Statement 1 and Statement 2 are Sanctuary
correct and Statement 2 is the correct (b) Bori and Papikonda Wildlife Sanctuaries
explanation for Statement 1 and Bandipur and Mukurthi National Park
(b) Both Statement 1 and Statement 2 are (c) Campbell Bay and Galathea National
correct but Statement 2 is not the correct Parks and Satkosia Reserve Forests
explanation for Statement 1 (d) Neyyar, Peppara and Shendurney
(c) Statement 1 is correct but Statement 2 is Wildlife sanctuaries and Nagarhole
not correct National Park
(d) Statement 1 is not correct and Statement
2 is correct
13
Vajiram & Ravi Prelims Test Series (2023)
Full Length Test No. – 01 (R4346)
_
87. Consider the following pairs: 90. The Government of India has developed
National Credit Framework (NCrF) to:
Sl. Regions Important reason (a) manage the balance sheet of the
No. for being in the Domestic Systemically Important Banks
news recently (b) assign credit-based value to different
1 Zaporizhzhia : Conflict between levels of learning
Russia and Ukraine (c) create a unified credit rating system for
2 Haiti : Political gridlock and infrastructure projects
rampant gang (d) increase the investments in
violence manufacturing sectors
3 Abidjan : United Nations
Convention to 91. With reference to Ancient Indian Literature,
Combat consider the following statements:
Desertification Sl. Treatise Author
(UNCCD) adopted a No.
global pledge to 1. Ashtadhyayi : Panini
boost drought 2. Mahabhasya : Jayaditya
resilience 3. Kasika Vritti : Kautilya

Which of the pairs given above are correctly Which of the pairs given above is/are
matched? correctly matched?
(a) 1 and 2 only a) 1 only
(b) 2 and 3 only b) 1 and 2 only
(c) 1 and 3 only c) 2 and 3 only
(d) 1, 2 and 3
d) 1 and 3 only
88. The “Schengen Zone”, sometimes seen in
the news, roughly corresponds to which one 92. Consider the following statements with
of the following regions? reference to the Ahoms of medieval India:
(a) Uranium rich regions in Central Asia 1. They were originally a Mongoloid tribe
(b) Area in Europe which allows free from north Burma.
movement of people without passports 2. Chaolung Sukapha was the founder of
(c) Countries surrounding the Black Sea the Ahom kingdom.
(d) Regions across the Baltic Sea from 3. They revolted against the British attempt
Russia to Germany to incorporate them into their dominion
after the First Burma War.
89. Consider the following statements: Which of the statements given above are
1. Mimicry is the ability of a species to correct?
resemble another species to gain an (a) 1 and 2 only
advantage in the surroundings. (b) 2 and 3 only
2. In butterflies, mimicry is reflected in the (c) 1 and 3 only
colour of wings as well as behaviour in (d) 1, 2 and 3
their flight.
Which of the statements given above is/are 93. “Bhagabat Chatuspathi’’ which was
correct?
dedicated to the study of Indian religion and
(a) 1 only
philosophy was founded by:
(b) 2 only
(a) Surendranath Banerjee
(c) Both 1 and 2
(b) Satishchandra Mukherjee
(d) Neither 1 nor 2
(c) Ananda Mohan Bose
(d) Rabindranath Tagore

14
Vajiram & Ravi Prelims Test Series (2023)
Full Length Test No. – 01 (R4346)
_
94. With reference to Medieval Indian History, 98. Consider the following pairs:
‘Bhagavad-Gita’ was translated into the
Persian language by: Sl. Institution Founder
(a) Babur No.
(b) Jehangir
(c) Dara Shikoh 1. Archaeological : Alexander
(d) Aurangzeb Survey of India Cunningham

2. Asiatic Society : William Jones


95. With reference to the history of India, the of Bengal
sites of Ukshi, Jambharun, Kasheli, and
Barsu are known for: 3. Calcutta : William
(a) chief ports of the Sangam age Medical Bentinck
(b) important trading centres of the College
Vijayanagara empire
(c) Buddhist pilgrimage centres How many of the above pairs is/are correctly
(d) prehistoric geoglyphs matched?
(a) None of the pairs
96. Which of the following sites in India have (b) Only one pair
Mural paintings? (c) Only two pairs
1. Ravan Chhaya Rock Shelter (d) All three pairs
2. Lepakshi Temple
99. With reference to colonial rule in India, the
3. Ajanta Caves Whitley Commission was appointed to:
4. Khajuraho Group of Monuments (a) examine the working of the Government
Select the correct answer using the code under the Montague-Chelmsford reforms
given below: (b) inquire into the conditions and prospects
(a) 1 and 3 only of universities in India
(b) 2 and 4 only (c) investigate the working conditions of
(c) 1, 2 and 3 only labour in British India
(d) 1, 3 and 4 only (d) look into problems of the land revenue
administration
97. The Changthangi goat is found in the cold,
100. Which of the following situations would
arid region surrounding the Ladakh region.
lead to the formation of fog in the
The breed is raised mainly for its ultra-fine atmosphere?
wool production. It is usually domesticated 1. When the air near the ground cools
and raised by nomadic communities called and stabilizes
the Changpa in Greater Ladakh. The 2. When moist air is cooled as it rises due
National Bureau of Animal Genetic to mountains
Resources has put the Changthangi goat 3. When cold air is blown over a hot
breed on the ‘endangered species list’. surface
The wool that is produced by Changthangi Select the correct answer using the code
goat is mostly used for? given below:
(a) 1 and 3 only
(a) Gadwal Sarees
(b) 1 and 2 only
(b) Chakhesang Shawls (c) 3 only
(c) Kinnauri Shawls (d) 2 and 3 only
(d) Pashmina shawls

15
Vajiram & Ravi Prelims Test Series (2023)
Full Length Test No. – 01 (R4346)
_
Prelims Full Length Test – 01 (R4346) - Answer Key

1. (c) 11. (b) 21. (d) 31. (b) 41. (c) 51. (c) 61. (b) 71. (d) 81. (d) 91. (a)

2. (a) 12. (b) 22. (c) 32. (d) 42. (c) 52. (b) 62. (b) 72. (d) 82. (d) 92. (d)

3. (d) 13. (a) 23. (a) 33. (c) 43. (c) 53. (b) 63. (d) 73. (a) 83. (c) 93. (b)

4. (c) 14. (b) 24. (c) 34. (a) 44. (c) 54. (b) 64. (b) 74. (c) 84. (a) 94. (c)

5. (a) 15. (c) 25. (b) 35. (c) 45. (a) 55. (b) 65. (d) 75. (b) 85. (b) 95. (d)

6. (b) 16. (a) 26. (c) 36. (a) 46. (a) 56. (b) 66. (c) 76. (d) 86. (a) 96. (c)

7. (c) 17. (b) 27. (d) 37. (b) 47. (a) 57. (c) 67. (c) 77. (b) 87. (d) 97. (d)

8. (a) 18. (a) 28. (b) 38. (d) 48. (c) 58. (d) 68. (c) 78. (d) 88. (b) 98. (d)

9. (a) 19. (b) 29. (c) 39. (c) 49. (c) 59. (d) 69. (b) 79. (b) 89. (c) 99. (c)

10. (c) 20. (a) 30. (c) 40. (d) 50. (a) 60. (c) 70. (d) 80. (d) 90. (b) 100. (b)

Q1.
Answer: c
Explanation:
● Nominal GDP (Gross Domestic Product) reflects the total value of goods and services
produced in a current year at current-year prices. On the other hand, real GDP tracks the
total value of goods and services by calculating the quantities at constant prices that are
adjusted for inflation. Currently, the base year for GDP calculation is 2011-12. Thus,
nominal GDP measures output using current prices, while real GDP measures output
using constant prices.
● Real GDP is different from nominal GDP, which does not account for inflation. This
means that if inflation is positive, real GDP will be lower than nominal, and vice versa.
Without a real GDP adjustment, positive inflation greatly inflates GDP in nominal terms.
Hence, in inflationary periods, real GDP will be lower than nominal GDP. In deflationary
times, real GDP will be higher. So, statements 1 and 2 are correct.
Therefore, option (c) is the correct answer.
_
Q2.
Answer: a
Explanation:
● When a country's Current Account Deficit (CAD) decreases, it typically means that the
country is importing fewer goods and services compared to what it is exporting. This
leads to less demand of dollar and a higher demand of rupee. This can lead to an
appreciation of the country's currency. Conversely, if a country's CAD increases, it can
lead to a depreciation of the country's currency. So, statement 1 is correct.
● Reverse Repo Rate is the interest rate at which the RBI borrows money from banks for
the short term. When the Reserve Bank of India (RBI) increases the Reverse Repo Rate,
it incentivises banks to deposit surplus funds with the RBI to earn higher interest. This
results in a decline in the money supply in the economy. Thus, the lendable resources of
the bank decreases and the cost of borrowing money increases. So, statement 2 is
correct.
● An increase in interest rates by the Federal Reserve, the Central Bank of the United States
of America, could potentially lead to a decrease in capital inflows in the Indian economy.
This is because an increase in interest rates in the United States may make investments
in the United States more attractive, causing investors to shift their capital from
emerging markets such as India back to the United States. So, statement 3 is not correct.
Therefore, option (a) is the correct answer.

Q3.
Answer: d
Explanation:
● Inflation is a sustained increase in the general price level of goods and services in an
economy over a period of time. Inflation can be caused by a variety of factors, including
supply-side factors such as rising costs of production, and demand-side factors such as an
increase in consumer spending.
● When the Reserve Bank of India (RBI) purchases government bonds and other securities
from banks, it injects money into the banking system and increases the money supply.
This increases the level of inflation in an economy. During inflation, which is a sustained
increase in the general price level of goods and services, the Reserve Bank of India (RBI)
sells government bonds and other securities to banks, this process is known as
Quantitative Tightening (QT). When the RBI sells government bonds and other securities
to banks, it withdraws money from the banking system and decreases the money supply
and inflation. So, statement 1 is correct.
● When the RBI sells dollars, it results in extinguishing an equivalent amount of rupees,
thus reducing the rupee liquidity in the market. This is most likely to lead to rupee
appreciation. So, statement 2 is correct.
● During inflation, the Reserve Bank of India (RBI) increases the benchmark interest rates
including the repo rate in order to curb inflationary pressures. An increase in the repo
rate makes borrowing more expensive for commercial banks, and this can discourage
_
borrowing and spending, which can help to decrease demand and lower prices. An
increase in the repo rate also leads to an increase in the cost of funds for banks, which in
turn leads to an increase in the lending rate. This can discourage borrowing and spending,
which can help to decrease demand and lower prices. So, statement 3 is correct.
Therefore, option (d) is the correct answer.

Q4.
Answer: c
Explanation:
● The G20 is a forum for international cooperation on financial and economic issues, and
is made up of 19 countries and the European Union. Within the G20, there are several
different tracks and groups that work on specific issues, including the Sherpa Track,
Finance Track, and Engagement Group.
● The Sherpa Track is responsible for coordinating and preparing the agenda for the G20
summit meetings, which take place annually. The Sherpa is typically a senior government
official or an expert in public life who is appointed by the G20 country's leader. The Sherpa
is responsible for communicating with other G20 members and coordinating the work of
various ministers and officials in the lead-up to the summit. So, statement 1 is correct.
● The Finance Track of G20 is responsible for issues related to finance and economic policy,
and it is led by the G20 Finance Ministers and Central Bank Governors. The Finance Track
focuses on issues such as international financial stability, monetary policy coordination,
and financial regulation. It also deals with global economic governance and economic
management in the wake of the global financial crisis. So, statement 2 is correct.
● The Engagement Group of G20 is an important track which brings together
representatives of civil society, the private sector, and academia with G20
representatives to discuss and provide recommendations on issues of common concern.
The engagement group focuses on enhancing cooperation between G20 and non-G20
stakeholders and in particular, seeks to involve representatives of the private sector and
civil society in the G20 process.
Therefore, option (c) is the correct answer.
Relevance: India assumed the presidency of the G20 grouping from Indonesia, on December 1,
2022

Q5.
Answer: a
Explanation:
● In India, the Sovereign Green Bonds are issued by the Reserve Bank of India. The bonds
are issued by any sovereign entity, inter-governmental groups or alliances and
corporations. The aim of the issuance of the bond is that the proceeds of the bonds are
utilised for projects classified as environmentally sustainable. The framework for the
Sovereign Green Bond was issued by the government on November 9, 2022. So,
statement 1 is correct.
_
● The Union Budget 2022-23 announced the issue of Sovereign Green Bonds in keeping
with the ambition to significantly reduce the carbon intensity of the economy. They will
be issued for mobilising resources for green infrastructure as a part of the government’s
overall market borrowings in 2022-23. The proceeds will be deployed in public sector
projects which help in reducing the carbon intensity of the economy. So, statement 2 is
correct.
● They will be available to foreign investors as well as Indian companies and
organisations. It will be rupee-denominated papers that will be open for foreign
investment through the Foreign Portfolio Investment (FPI) route. They will be eligible for
trading in the secondary market and will be designated as specified securities under the
‘Fully Accessible Route’ for investment by non-residents. So, statement 3 is not correct.
Therefore, option (a) is the correct answer.
Relevance: The RBI has planned to issue Sovereign Green Bonds worth Rs 16,000 crore.

Q6.
Answer: b
Explanation:
• The online gaming industry in India has been growing rapidly in recent years. The majority
of the revenue in the Indian online gaming industry comes from mobile games, followed
by PC and console games.
● In India, 100% Foreign Direct Investment (FDI) is allowed under the automatic route for
online gaming which falls under the electronic system, IT sectors. This means that
approval from the appropriate ministries is not required for such investments. So,
statement 1 is not correct.
● In India, a flat Goods and Services Tax (GST) rate of 28% is applicable on all online games,
including games of skills and chance. The rate will be chargeable on ‘gross revenue’ basis
that is, on the total stake value and not on the platform fee paid by the user. So,
statement 2 is correct.
● The Ministry of Electronics and Information Technology (MeitY) has been appointed as
the nodal ministry for online gaming in India. It will make rules regarding the Online
Gaming Industry in India. So, statement 3 is correct.
Therefore, option (b) is the correct answer.

Q7.
Answer: c
Explanation:
● Goods and Services Tax (GST) is a value-added tax levied on most goods and services sold
for domestic consumption. It is an indirect tax used in India on the supply of goods and
services. Items that are not covered under GST are called a negative list. These items are
notified under Schedule III of the Act.
● The following items are on the negative list under GST: Services by an employee to the
employer in the course/ relation to employment; Services of funeral, burial,
crematorium or mortuary; Sale of land; Sale of completed buildings; Actionable claims
_
(other than lottery, betting and gambling); Services by any court or Tribunal; Functions
performed by the MPs, MLAs etc.; Duties performed by any person who holds any post in
pursuance of the provisions of the Constitution in that capacity. So, point 1 is correct.
● Recently, the GST council meeting held in 2022 made a decision that GST exemption on
the transport of passengers by air to and from north-eastern states will be restricted to
economy class only. A levy of 12 per cent GST will apply for Business class travel from
airports in the northeastern states. So, point 2 is not correct.
● The Government recently clarified that fresh milk and pasteurised milk are fully
exempted from GST. Further, milk products like curd, lassi, buttermilk and paneer are
also exempted from GST if sold in forms other than those pre-packaged and labelled. A
nominal GST of 5 per cent applies to curd, lassi, buttermilk and paneer when sold in pre-
packaged and labelled form and Ultra High-Temperature Milk. So, point 3 is not correct.
● Transmission or distribution of electricity by an electricity transmission or distribution
utility is exempted under the purview of GST. Before GST, electricity supply was
exempted under the respective State’s Value Added Tax Acts. Also, services provided by
an electricity transmission or distribution utility by transmission or distribution of
electricity were exempted under the Service Tax. So, point 4 is correct.
Therefore, option (c) is the correct answer.

Q8.
Answer: a
Explanation:
● A yield curve is a graph that shows the relationship between bond yields (Yield is a
measure of the return on investment) and maturity dates for bonds of similar credit
quality. It plots the yields of bonds with different maturity dates, such as 3-month, 2-year,
5-year, 10-year, and 30-year bonds.
● A positively sloped yield curve is seen as a sign of a healthy economy, as it indicates that
long-term economic growth is expected and that investors are willing to accept lower
yields for longer-term investments.
● A downward-sloping or inverted yield curve shows that markets expect the economy to
slow down and interest rates to drop in the future. When markets fear an economic
slowdown, they expect the demand for money to go down, which will drive interest rates
lower. They also, thus, expect inflation to remain lower. Lower spreads, which is the
difference between short-term and long-term yields, therefore result in a downward-
sloping yield curve.
Therefore, option (a) is the correct answer.

Q9.
Answer: a
Explanation:
● According to the Securities and Exchange Board of India (SEBI), an Open Offer is made
by a company that is acquiring shares, to the shareholders of the target company, inviting
them to sell their shares at a particular price. The purpose of an open offer is to provide
_
an exit option to the company’s shareholders since there is a change in control or
substantial acquisition of shares. So, statement 1 is correct.
● It is initiated if an acquirer holds more than 25 per cent of the public shareholding in the
company. Prior to 2011, an open offer got triggered if an acquirer owned more than 15
per cent of the public shareholding in a company. So, statement 2 is not correct.
● According to SEBI rules, the open offer price for the acquisition of shares under the
minimum open offer shall be:
○ the highest negotiated price under the share purchase agreement triggering the
offer; and
○ the volume-weighted average price of shares acquired by the acquirer during 52
weeks preceding the public announcement; and
○ the highest price paid for any acquisition by the acquirer during 26 weeks
immediately preceding the PA (Purchase Agreement); and
○ the volume-weighted average market price for sixty trading days preceding the PA
● With respect to the above conditions, the price offered in an open offer can be less than
the market price of the shares at the time the offer is made. So, statement 3 is not
correct.
Therefore, option (a) is the correct answer.
Relevance: Adani Group acquired a 29.18 per cent stake in television channel NDTV Ltd through
an open offer.

Q10.
Answer: c
Explanation:
● The Union Budget of 2022-23 had announced that surety bonds will be accepted as a
substitute for bank guarantees in government procurements tenders. This was done
with the intent to reduce indirect costs for suppliers and work contractors. The premium
charged for this surety bond is expected to be kept lower to make the product affordable.
So, statement 1 is correct.
● Surety Bond is a three-party agreement that legally binds together a principal who needs
the bond, an obligee who requires the bond and a surety company that sells the bond.
Surety bonds provide a financial guarantee that contracts will be completed according to
pre-defined and mutual terms.
● Surety bonds are aimed at infrastructure development to reduce indirect costs for
suppliers and work contractors, thereby diversifying their options and acting as a
substitute for bank guarantees. The bond is provided by the insurance company on
behalf of the contractor to the entity which is awarding the project. So, statement 2 is
correct.
Therefore, option (c) is the correct answer.
Relevance: India launched its first-ever surety bond insurance product in December 2022.

Q11.
Answer: b
Explanation:
_
● The Constitution of India distributes core functions of governance amongst the three
organs of a government — the legislature (to enact laws), the executive (to enforce the
laws and to manage day-to-day administration), and the judiciary (to adjudicate disputes).
This distribution is based on the doctrine of separation of powers proposed by the 18th
Century French philosopher Montesquieu.
● The Doctrine of Checks and Balances establishes a system of checks and balances among
the different branches of government so that no one branch can dominate the others.
This ensures that the powers of government are balanced and that the different branches
are able to hold each other accountable.
● Checks and balances between the three organs are ensured through:
○ The power of the Judiciary to exercise judicial review over legislative and
executive actions (Article 13). So, point 3 is correct.
○ Appointment of judges by the Executive head and removal of judges on the basis
of a resolution passed by the Parliament (Article 124). So, point 2 is correct.
○ The judiciary is bound by the procedure established by law in adjudication on
question of law (Article 145).
○ Parliamentary form of Government where the executive is responsible to the
legislature (Article 74).
● The provisions of the Constitution that provide for the separation of functions and
powers between the three organs of Government are
○ Articles 74 and 163 restrict the courts from inquiring into the advice tendered by
the Council of Ministers to the President and the Governor.
○ Articles 121 and 211 restrict the Parliament and the State Legislature from
discussing the Judicial conduct of a judge of the Supreme Court and the High
Courts unless the resolution of removal of the judge is under consideration. So,
point 1 is not correct.
○ Article 361 provides immunity to the President or the Governor from being
answerable to any court for the exercise and performance of the powers and
duties of his office.
Therefore, option (b) is the correct answer.

Q12.
Answer: b
Explanation:
● In the Kesavananda Bharati Case (1973), the Supreme Court held that the Parliament can
amend the Constitution including Fundamental Rights but it cannot change the basic
structure of the Constitution. Thus, it laid down a new doctrine of the ‘basic structure’ of
the Constitution. However, the Supreme Court is yet to define or clarify what constitutes
the ‘basic structure’ of the Constitution. So, statement 1 is not correct.
● Following are some of the important elements of the ‘basic structure’ of the Constitution:
○ Supremacy of the Constitution
○ Limited power of Parliament to amend the Constitution. So, statement 2 is
correct.
○ Sovereign, democratic and republican nature of the Indian polity
_
○ Secular character of the Constitution
○ Separation of powers between the legislature, the executive and the judiciary
○ Federal character of the Constitution
Therefore, option (b) is the correct answer.

Q13.
Answer: a
Explanation:
● In the Indian Parliament, a "Money Bill" is a type of legislation that deals specifically with
the government's finances, including matters such as taxes, government spending, and
government borrowing. Article 110 of the Constitution deals with the definition of money
bills.
● A Money Bill can only be introduced in the Lok Sabha (the lower house of the Indian
Parliament), and it must be recommended by the President before it can be introduced.
Once a Money Bill is passed by the Lok Sabha, it is sent to the Rajya Sabha (the upper
house of the Indian Parliament) for review. The Rajya Sabha can make recommendations
on a Money Bill, but the Lok Sabha is not obligated to accept them, and the Rajya Sabha
cannot reject or amend a Money Bill. So, statement 1 is correct.
● A Money Bill can be sent for the President's assent even if it is not approved by the Rajya
Sabha. There is no provision of joint sitting of both the Houses in this regard. So,
statement 2 is not correct.
● The President of India does have discretion in case of a money bill. It can be rejected or
approved but cannot be returned for reconsideration by the President. So, statement 3
is not correct.
Therefore, option (a) is the correct answer.
Knowledge Box

Money Bills:
● It can be introduced only by a Minister.
● It can be detained by the Rajya Sabha for a maximum period of 14 days only.
● It requires the certification of the Speaker when transmitted to the Rajya Sabha.
● Its defeat in the Lok Sabha leads to the resignation of the Government.

Q14.
Answer: b
Explanation:
● A Constitution bench of the Supreme Court has said that the Fundamental Rights
mentioned in Article 19 and Article 21 of the Constitution are enforceable even against
private persons, and not just the State and its instruments. This puts an obligation on the
State to ensure that private entities also abide by Constitutional norms.
_
● The Supreme Court held that the Right to Free Speech and Expression guaranteed under
Article 19(1)(a) cannot be curbed by any additional grounds other than those already
laid down in Article 19(2).
● Under Article 19 (2), the State can impose reasonable restrictions on the grounds of the
sovereignty and integrity of India, security of the State, friendly relations with foreign
States, public order, decency or morality, contempt of court, defamation, and incitement
to an offence.
● Protection of six rights under Article 19 regarding freedom of (i) Speech and Expression,
(ii) Assembly, (iii) Association, (iv) Movement, (v) Residence, and (vi) Profession is
available only to citizens and not to foreigners.
● Rights under Article 19 of the Constitution of India include freedom to seek, receive, and
impart information and ideas of all kinds, regardless of frontiers either orally or in the
form of writing or print, in the form of art, or through any other media including social
media.
Therefore, option (b) is the answer.
Relevance: Recently, the Supreme Court has ruled that Articles 19(1) and 21 can be enforced
against private entities.

Q15.
Answer: c
Explanation:
● The Directorate of Enforcement (ED) is a multi-disciplinary organisation mandated with
investigation of economic crimes and violations of foreign exchange laws.
● It is an agency under the Ministry of Finance and works closely with other law
enforcement agencies, such as the Central Bureau of Investigation (CBI), the Income Tax
Department, and the Directorate of Revenue Intelligence (DRI), to investigate and
prosecute economic crimes.
● The statutory functions of the Directorate include enforcement of following Acts:
○ Prevention of Money Laundering Act, 2002 (PMLA): ED has been given the
responsibility to enforce the provisions of the PMLA by conducting investigation
to trace the assets derived from proceeds of crime, to provisionally attach the
property and to ensure prosecution of the offenders and confiscation of the
property by the Special court. So, point 1 is correct.
○ Foreign Exchange Management Act, 1999 (FEMA): ED has been given the
responsibility to conduct investigation into suspected contraventions of foreign
exchange laws and regulations, to adjudicate and impose penalties on those
adjudged to have contravened the law. So, point 2 is correct.
○ Fugitive Economic Offenders Act, 2018 (FEOA): This law was enacted to deter
economic offenders from evading the process of Indian law by remaining outside
the jurisdiction of Indian courts. It is a law whereby the Directorate is mandated
to attach the properties of the fugitive economic offenders who have escaped
from India warranting arrest and provide for the confiscation of their properties
to the Central Government. So, point 4 is correct.
_
● ED is not responsible for conducting investigations of offences under the Prevention of
Corruption Act, 1988 and Customs Tariff Act, 1975. So, point 3 is not correct.
○ The Prevention of Corruption Act, 1988 was enacted to prevent corruption in the
offices of a public servant.
○ The Customs Tariff Act, 1975 (CTA) provides for the levy and collection of customs
duties on imported and exported goods. It is administered by the Central Board
of Indirect Taxes and Customs (CBIC), Ministry of Finance.
Therefore, option (c) is the correct answer.
Knowledge Box
● Sponsoring agency under COFEPOSA: Under the Conservation of Foreign Exchange
and Prevention of Smuggling Activities Act, 1974 (COFEPOSA), ED is empowered to
sponsor cases of preventive detention with regard to contraventions of FEMA.

Q16.
Answer: a
Explanation:
● The Memorandum of Procedure (MoP) is the document governing the process of
appointment of judges and appointment of the Chief Justice of India (CJI).
● As per this, appointment to the office of the Chief Justice of India should be of the senior
most Judge of the Supreme Court considered fit to hold the office. The Union Ministry
of Law and Justice would, at the appropriate time, seek the recommendation of the
outgoing Chief Justice of India for the appointment of the next Chief Justice of India. After
receipt of the recommendation of the Chief Justice of India, the Union Minister of Law &
Justice will put up the recommendation to the Prime Minister who will advise the
President in the matter of appointment. So, statement 1 is correct.
● Although the MoP says that the CJI’s views must be sought “at the appropriate time” it
does not specify a timeline for the process. It normally takes place a month before the
retirement of the incumbent CJI. The MoP also does not prescribe a timeline for the
Centre to forward the recommendations sent by the Supreme Court collegium. So,
statement 2 is not correct.
Therefore, option (a) is the correct answer.
Relevance: The five-judge Supreme Court collegium, headed by the Chief Justice of India
discussed the process of appointment of the next Chief Justice.

Q17.
Answer: b
Explanation:
● Delimitation is the act of redrawing boundaries of Lok Sabha and State Assembly seats to
represent changes in population. The main objective of delimitation is to provide equal
representation to equal segments of a population.
● Delimitation is carried out by an independent Delimitation Commission under provisions
of the Delimitation Commission Act. The Delimitation Commission is appointed by the
President of India and it works in collaboration with the Election Commission of India.
_
● Delimitation Commission is composed of the following: a retired Supreme Court judge,
the Chief Election Commissioner of India and respective State Election Commissioners.
The Delimitation Commission is to work without any executive influence. So, statement
1 is not correct.
● The Delimitation Commission is supposed to determine the number and boundaries of
constituencies in a way that the population of all seats, so far as practicable, is the same.
The Commission is also tasked with identifying seats reserved for Scheduled Castes and
Scheduled Tribes for Parliamentary and State Assembly elections. So, statement 2 is
correct.
● The orders of the Delimitation Commission come into force on a date to be specified by
the President of India on its behalf. The copies of its orders are laid before the House of
the People and the State Legislative Assembly concerned, but no modifications are
permissible therein by them. The Constitution mandates that the Commission’s orders
are final and cannot be questioned before any court as it would hold up an election
indefinitely. So, statement 3 is not correct.
Therefore, option (b) is the correct answer.
Relevance: Election Commission recently began the ‘delimitation’ exercise in Assam.

Q18.
Answer: a
Explanation:
● Under Articles 155 and 156 of the Constitution of India, a Governor is appointed by the
President and holds office “during the pleasure of the President”. If this pleasure is
withdrawn before the completion of the five-year term, the Governor has to step down.
As the President works on the aid and advice of the Prime Minister and the Council of
Ministers, in effect, the Governor can be appointed and removed by the Central
Government.
● The Governor holds office “at the pleasure of the President”. In BP Singhal vs Union of
India (2010), the Supreme Court elaborated on the pleasure doctrine. It upheld that “no
limitations or restrictions are placed on the ‘at pleasure’ doctrine”. In its judgement, the
Bench, while noting that the President can remove the Governor from office “at any
time without assigning any reason and without giving any opportunity to show cause”,
the power to remove can’t be exercised in an “arbitrary, capricious or unreasonable
manner”. So, statement 1 is correct.
● In BP Singhal vs Union of India (2010) case, the Supreme Court held that the Governor
cannot be removed on the ground that he is out of sync with the policies and ideologies
of the Union Government or the party in power at the Centre. Nor can he be removed on
the ground that the Union Government has lost confidence in him. The Bench held that
the court will presume that the President had “compelling and valid” reasons for the
removal but if a sacked Governor comes to the court, the Centre will have to justify its
decision. So, statement 2 is not correct.
● The Sarkaria Commission had recommended that Governors are not sacked before
completing their five-year tenure, except in “rare and compelling” circumstances.
Sarkaria Commission did not recommend incorporating a provision to impeach the
_
Governor by the State Legislative Assembly as it may not ensure objectivity and
impartiality. So, statement 3 is not correct.
Therefore, option (a) is the correct answer.

Q19.
Answer: b
Explanation:
● The term ‘equality’ as envisaged in the Constitution of India means the absence of special
privileges to any section of the society and the provision of adequate opportunities for
all individuals without any discrimination. The Preamble secures to all citizens of India
equality of status and opportunity. This provision embraces three dimensions of
equality–civic, political and economic.
● Equality in India does not mean that everyone is treated exactly the same. Instead, it
means that everyone is treated fairly and with equal rights and opportunities, regardless
of their individual characteristics or differences. This can involve taking steps to address
and correct past and present imbalances and discrimination, and ensuring that all
individuals are able to fully participate in and contribute to society.
● Equality in India does not mean that everyone must have the same level of income and
wealth. Instead, it means that everyone is entitled to equal rights and opportunities, and
that social and economic policies should be designed to promote fairness and reduce
disparities.
● Uniform Civil Code is mentioned in the Directive Principles of State Policy and is not
Therefore, option (b) is the correct answer.
Knowledge Box

● Civic equality is ensured by following Fundamental Rights:


○ The State shall not deny to any person equality before the law or equal
protection of the laws (Article 14)
○ The State shall not discriminate against any citizen on the ground of religion
(Article 15)
○ Equality of opportunity for all citizens in matters of public employment (Article
16)
○ Abolition of untouchability (Article 17)
○ Abolition of titles (Article 18)
● Following two provisions in the Constitution provides political equality:
○ Article 325: No person is to be declared ineligible for inclusion in electoral rolls
on grounds of religion, race, caste or sex.
○ Article 326: Elections to the Lok Sabha and the state assemblies to be on the
basis of adult suffrage.
● Economic equality: The Directive Principles of State Policy (Article 39) secures to men
and women equal right to an adequate means of livelihood and equal pay for equal
work.
_
Q20.
Answer: a
Explanation:
● The idea of the ‘Doctrine of Proportionality’ appears first in Aristotle's Nicomachean
Ethics. Aristotle's concept, in short, can be said to be of 'recta ratio', that is, the Right
reason or the right relationship between the state and the citizen, adjudicated by the
rule of law.
● The doctrine finds its place in Administrative Law and is used at the stage of Judicial
Review. The doctrine asserts that there must be a reasonable balance between the
desired result and the measures taken to reach that goal. The action taken must not be
disproportionate to the consciousness of the court and the said action can then be
challenged by way of judicial review.
● Proportionality means that the administrative action should not be more drastic than it
ought to be for obtaining the desired result.
Therefore, option (a) is the correct answer.
Relevance: Recently, the Supreme Court accepted that the policy of demonetisation satisfies the
doctrine of proportionality.

Q21.
Answer: d
Explanation:
● The earliest evidence of millets in India is found in the Indus Valley Civilization (IVC).
Rice, Bajra, and Millets found from many IVC sites showed evidence of cultivation. So,
statement 1 is correct.
● A number of small-seeded annual grasses that are grown primarily on marginal lands in
dry areas of temperate, subtropical, and tropical climates are collectively referred to as
millet. Ragi (finger millet), Jowar (sorghum), Sama (little millet), Bajra (pearl millet), and
Variga are some of the popular millets available in India (Proso millet). So, statement 2 is
correct.
● Millets are less expensive and more nutritious than wheat and rice because of their high
content of protein, fibre, vitamins, and minerals like iron. Calcium and magnesium are
high in millets. Among all the food grains, ragi is known to have the highest calcium
content.
● Millet is a naturally gluten-free grain that can be used as a whole seed. Millets can aid in
addressing a variety of health issues like obesity and diabetes due to their lack of gluten
and low glycaemic index. So, statement 3 is correct.
● Millets don't need a specific photoperiod to flower. They are resistant to climate change
and photo-insensitive. Millets can grow on deficient soils with little to no outside
assistance. So, statement 4 is correct.
Therefore, option (d) is the correct answer.
Relevance: In preparation for the International Year of Millets (IYM) in 2023, the Ministry of
Agriculture and Farmers Welfare has planned a number of pre-launch activities.
_
Q22.
Answer: c
Explanation:
● Regenerative Agriculture focuses on improving the health of the soil which has been
degraded by the use of heavy machinery, fertilisers and pesticides in intensive farming. It
can improve crop yields (the volume of crops produced) by improving the health of the
soil and its ability to retain water, as well as reducing soil erosion. So, statement 1 is
correct.
● The methods included in this farming are minimising the ploughing of land, and cover
cropping. Monoculture is not a method of regenerative agriculture. Instead, regenerative
farmers choose crops to plant together (this is called 'intercropping') based on how well
they will each support each other, as well as the greater ecosystem on the farm. So,
statement 2 is not correct.
● Regenerative farmers and ranchers make every effort to reduce their reliance on
synthetic inputs, such as herbicides, pesticides, and chemical fertilizers. In the process of
prioritizing soil health, many growers naturally use fewer chemical inputs. Instead, as
beneficial insects and wildlife return and diverse crop and livestock rotations disrupt
weed cycles, the ecosystem becomes more resilient. So, statement 3 is correct.
Therefore, option (c) is the correct answer.

Q23.
Answer: a
Explanation:
● Stargazing is the practice of observing celestial bodies, such as stars, planets, galaxies,
and other astronomical objects, in the night sky. It is often done using the naked eye, but
telescopes, binoculars, and other optical devices can also be used to enhance the viewing
experience. Stargazing spots are dependent on a number of conditions that provide
optimal viewing opportunities for celestial bodies like low light pollution, clear and stable
weather, high altitude, dark sky parks and reserves, remote locations, such as deserts or
mountains. Some ideal stargazing spots in India include:
● Katao, a small village in the North Sikkim district of the Indian state of Sikkim, is a suitable
spot for stargazing due to its high altitude and remote location. The village is situated at
an altitude of around 13,800 feet and is surrounded by snow-capped mountains and
rugged terrain, which can provide a clear and unobstructed view of the night sky. Katao
is far from the major cities and has low levels of light pollution which makes it a good
stargazing spot. So, point 1 is correct.
● Churu, a city located in the desert region of Rajasthan, India, is a suitable spot for
stargazing due to its remote location and low levels of light pollution. The city is located
far from major urban areas, which can help to reduce light pollution and provide a clear
view of the night sky. Additionally, being in the desert, Churu can have clear skies most of
the time, which could make it a good spot for stargazing. So, point 2 is correct.
_
● Burnpur, a city located in the state of West Bengal, India, is not a suitable spot for
stargazing due to its heavy industrialization, which can cause high levels of light
pollution and make it difficult to see stars in the night sky. The city is home to multiple
steel plants and factories, which use bright lights and machinery that can create a lot of
light pollution, obscuring the view of the night sky. So, point 3 is not correct.
● Spiti Valley is a high-altitude valley located in the Indian state of Himachal Pradesh,
known for its rugged and remote terrain, which makes it a suitable spot for stargazing.
The valley is situated at an altitude of around 12,500 feet and is surrounded by snow-
capped mountains, which can provide a clear and unobstructed view of the night sky.
Being in a remote location, it has low levels of light pollution, which can make it ideal for
stargazing. So, point 4 is correct.
● Kovalam is a coastal town located in the Indian state of Kerala, known for its beautiful
beaches and tropical climate. While Kovalam is a popular tourist destination, it is not the
most suitable spot for stargazing due to its proximity to the sea and the resulting high
levels of humidity in the air. The humidity can cause haze and obscure views of the stars.
So, point 5 is not correct.
Therefore, option (a) is the correct answer.
Knowledge Box

● The International Dark Sky Week (IDSW) is an annual event that takes place during the
week of the new moon nearest to Earth Day, which is April 22nd.
● The event is celebrated around the world and is organized by the International Dark-
Sky Association (IDA).
● IDA is a non-profit organization that works to protect the night skies for present and
future generations by advocating for responsible outdoor lighting and dark sky
preservation.
● The purpose of International Dark Sky Week (IDSW) is to raise awareness about the
issue of light pollution and its impacts on the environment, wildlife, and human health,
and to encourage individuals, communities, and organizations to take actions to reduce
light pollution.

Q24.
Answer: c
Explanation:
● The Brahmaputra is a major transboundary river in Asia, which flows through China,
India, and Bangladesh. The Brahmaputra is known for its powerful currents and frequent
_
floods, which can cause significant
damage to the surrounding areas. It
is also an important source of
irrigation and hydroelectric power
for the region.
● The drainage area lying in India is
194413 sq.km which is nearly 5.9%
of the total geographical area of the
country. It is bounded on the north
by the Himalayas, on the east by the
Patkari range of hills running along
the Assam-Burma border, on the
south by the Assam range of hills and
on the west by the Himalayas and the
ridge; separating it from Ganga sub-
basin. The sub-basin lies in the States of Arunachal Pradesh, Assam, Nagaland,
Meghalaya, West Bengal and Sikkim. So, points 1, 3, 4 and 5 are correct.
● Tripura is not part of the drainage basin of the Brahmaputra River. So, point 2 is not
correct.
Therefore, option (c) is the correct answer.
Knowledge Box

● Main North Bank Tributaries of Brahmaputra: Subansiri, Ronganadi, Dikrong, Buroi,


Borgong, Jiabharali, Dhansiri (North) Puthimari, Manas, Beki, Aie, Sonkosh
● Main South Bank Tributaries of Brahmaputra: Noadehing, Buridehing, Desang,
Dikhow, Bhogdoi, Dhansiri (South), Kopilli, Kulsi, Krishnai, Dhdhnoi, Jinjiran
● The characteristics of the north bank tributaries are different than that of the
south bank tributaries, which may be summarized as below:
● The North Bank Tributaries: Have very steep slopes and shallow braided channels
for a considerable distance from the foothills and in some cases right up to the
outfall. Have boulder, pebble and coarse sandy beds and carry a heavy silt charge.
Generally, have flashy floods.
● The South bank Tributaries: Have comparatively flatter grades and deep
meandering channels almost from the foothills. Have comparatively low silt charge.

Q25.
Answer: b
Explanation:
● Bhimtal Lake is located in Uttarakhand. There is an island in the centre of the lake, which
is home to the Bhimtal Lake Aquarium. So, pair 1 is not correctly matched.
● Rewalsar Lake, also known as Tso Pema, is located in Himachal Pradesh. It is considered
sacred by Hindus, Buddhists, and Sikhs. It is believed that the lake was created by the
_
Hindu God Lord Shiva and is named after the sage Lomas, who is said to have meditated
on an island in the lake. So, pair 2 is not correctly matched.
● Tso Kar Lake, also known as Tsho Kar, is in the Changthang region of Ladakh. It is a
saltwater lake and is home to a variety of wildlife, including migratory birds such as
flamingos and bar-headed geese. So, pair 3 is correctly matched.
● Gurudongmar Lake is a high-altitude lake located in Sikkim. It is considered sacred by
Buddhists and Hindus. The lake is named after Guru Padmasambhava, a Tibetan Buddhist
teacher. It is only accessible for about six months of the year due to heavy snowfall. So,
pair 4 is correctly matched.
Therefore, option (b) is the correct answer.

Q26.
Answer: c
Explanation:
● Ukraine is located in eastern Europe and is
the second largest country on the continent
after Russia. Its capital is Kyiv, which is
located on the Dnieper River in north-central
Ukraine.
● Ukraine is bordered by
○ Belarus to the north,
○ Russia to the east,
○ the Sea of Azov and the Black Sea to
the south,
○ Moldova and Romania to the
southwest,
○ and Hungary, Slovakia, and Poland to
the west. So, points 1, 3 and 5 are
correct.
● In the far southeast, Ukraine is separated from Russia by the Kerch Strait, which connects
the Sea of Azov to the Black Sea.
● Estonia and Czech Republic do not border with Ukraine. So, points 2 and 4 are not correct.
Therefore, option (c) is the correct answer.
Relevance: Russia recognized two independent republics out of Ukraine Donetsk and Luhansk -
Donbass region

Q27.
Answer: d
Explanation:
● The Western Disturbances are extratropical or subtropical cyclones that originate in the
Mediterranean region of Subtropical High. It provides winter rain to the Indian
_
subcontinent. It is a low-pressure depression or trough over the surface or upper air in
the westerly winds’ regime. So, statement 1 is correct.
● Low pressure usually forms over the Mediterranean Sea and the Caspian Sea and moves
across Iran, Iraq, Afghanistan, and Pakistan before arriving in India, where it is filled with
moisture. These moisture-laden western disturbances finally collide with the Himalayas
and become blocked; as a result, moisture is trapped, and precipitation is distributed
across North and Northwest India.
● Jet streams are narrow belts of high altitude (above 12,000 m) westerly winds in the
troposphere. These jet streams are located approximately over 27°-30° north latitude,
therefore, they are known as subtropical westerly jet streams. Over India, these jet
streams blow South of the Himalayas, all through the year except in summer. The
western cyclonic disturbances experienced in the north and north-western parts of the
country are brought in by this westerly flow. So, statement 2 is correct.
● A prevailing intense western disturbance is deep enough to spark rain or thundershowers
over parts of South India, apart from the familiar tracks in North-West, Central and East
India. A few/isolated places over Tamil Nadu, Puducherry, Karaikal and Kerala receives
showers while isolated places over Jammu & Kashmir, Himachal Pradesh, Uttarakhand,
Punjab, Haryana, Chandigarh and Uttar Pradesh witnesses snow/rain as a slow-moving
western disturbance extend its influence over regional weather. The snowfall and cold
snaps in hilly regions, as well as the plains, benefit rabi crops such as wheat and fruits
crops such as apples. So, statement 3 is correct.
Therefore, option (d) is the correct answer.

Q28.
Answer: b
Explanation:
● Steppes are flat grassland landscapes found in moderate climates. Steppe climate falls in
between the tropical and polar regions. In the northern hemisphere, the grasslands are
far more extensive and are entirely continental. In Eurasia, they are called the steppes.
So, statement 1 is not correct.
● On the eastern slopes of the Rockies, Chinook, a katabatic local wind similar to the Fohn
in Switzerland, flows. It comes with the depressions in winter from the Pacific coast
ascending the Rockies and then descending to the Prairies (katabatic wind). It is a hot
wind and may raise the temperature by 5 °C within a matter of 20 minutes. It melts the
snow-covered pastures and benefits agriculture and animal ranching. So, statement 2 is
correct.
● The temperate grasslands are ideal for extensive wheat cultivation. The cool, moist spring
stimulates early growth and the light showers in the ripening period help to swell the
grains to ensure a good yield. The warm, sunny summer is not only advantageous for
harvesting, but also enables the straw to be dried for farm use. In addition, the levelness
of the Steppes and other temperate grasslands all over the world makes ploughing and
harvesting a comparatively easy job. In the Prairies, the Argentinian Pampas, the
Ukrainian Steppes and the Downs of Australia, agriculture is completely mechanised. So,
statement 3 is correct.
_
Therefore, option (b) is the correct answer.

Q29.
Answer: c
Explanation:
● An equinox is an event in which a planet’s subsolar point passes through its Equator. The
equinoxes are the only time when both the Northern and Southern Hemispheres
experience roughly equal amounts of daytime and night-time.
● Solar declination describes the latitude of the Earth where the Sun is directly overhead at
noon. So, equinoxes are the only times of the year when the subsolar point is directly on
the Equator. Solar declination is 0° during the equinoxes.
● The subsolar point is an area where the Sun's rays shine perpendicular to the Earth's
surface at a right angle. The perceived centre of the Sun’s disk is in the same plane as
the Equator, only during an equinox when the Earth's 23.5° axis is not tilting toward or
away from the sun.
● After the March equinox, the subsolar point migrates north as the Northern Hemisphere
tilts toward the sun. Around June 21, the subsolar point hits the Tropic of Cancer,
(23.5°N). This is the June solstice.
● After the September equinox, the subsolar point continues to move south as the Southern
Hemisphere tilts toward the sun. Around December 21, the subsolar point hits the Tropic
of Capricorn (23.5°S). This is the December solstice.
● On January 3, the Perihelion is the time when the Earth is closest to the Sun and the
Aphelion is the time when the Earth is farthest from the Sun. The average distance of the
Earth from the Sun over a one-year period is about 149.6 million km.
Therefore, option (c) is the correct answer.

Q30.
Answer: c
Explanation:
● Simoon is located in Sahara and Arabian Desert. This is a very hot, dry, suffocating and
dust-laden wind that blasts across the African deserts, notably in Arabia, Syria, and
neighbouring countries generated by the extreme heat of the parched deserts or sandy
plains. So, pair 1 is not correctly matched.
● Sirocco is a warm, humid wind occurring over the northern Mediterranean Sea and
southern Europe. The Sirocco causes dusty dry conditions along the northern coast of
Africa, storms in the Mediterranean Sea, and cool wet weather in Europe. So, pair 2 is
correctly matched.
● Bora is a cold and typically very dry and often gusty katabatic wind from northeast in the
Adriatic coast of the Balkans. So, pair 3 is not correctly matched.
● Pampero is the name for a severe cold line squall that occurs over the Pampas of
Argentina and Uruguay. So, pair 4 is correctly matched.
Therefore, option (c) is the correct answer.

Q31.
_
Answer: b
Explanation:
● Digital Public Goods are digital resources that are freely available to everyone to use,
reuse, and redistribute without any restrictions. Some examples of digital public goods
include open-source software, open-access research papers, and open data sets. These
resources are considered to be public goods because they provide benefits to society as
a whole, rather than just to a specific group or individual.
○ Unified Payments Interface (UPI) is a system that powers multiple bank accounts
into a single mobile application (of any participating bank), merging several
banking features, seamless fund routing & merchant payments into one hood. UPI
is a digital public good that boosts productivity for the economy and the broader
population significantly. So, point 1 is correct.
○ Aarogya Setu is a mobile application developed by the Government of India which
connects the various essential health services with the people of India. It is a
digital public good as the stated purpose of this app is to spread awareness of
COVID-19 and to connect essential COVID-19-related health services to the people
of India. So, point 2 is correct.
○ Core Banking Solution is the networking of bank branches, which allows
customers to manage their accounts, and use various banking facilities from any
part of the world. It is not a digital public good as banks charge or levy fees to
use these services. So, point 3 is not correct.
○ Open Network for Digital Commerce (ONDC) is a non-profit company whose
network will enable the display of products and services from all participating e-
commerce platforms in search results across all apps on the network. It is based
on open-sourced methodology, using open specifications and open network
protocols, and is independent of any specific platform. It is a digital public good.
So, point 4 is correct.
○ Aadhaar is a 12-digit unique identity number that can be obtained voluntarily by
the citizens of India and resident foreign nationals who have spent over 182 days
in twelve months immediately preceding the date of application for enrolment,
based on their biometric and demographic data. It is a digital public good that has
ensured faster delivery of services. So, point 5 is correct.
Therefore, option (b) is the correct answer.

Q32.
Answer: d
Explanation:
● Gene therapy is an experimental technique to treat genetic diseases. It is based on the
introduction of nucleic acid with the help of a vector, into a diseased cell or tissue, to
correct the gene expression and thus prevent, halt, or reverse a pathological process.
Both viral and nonviral vectors can be used to deliver the correct gene.
● Applications of Gene Therapy:
o Gene therapies are being developed to treat several different types of inherited
blindness where patients gradually lose the light-sensing cells in their eyes. In one
_
small trial of patients with a form of degenerative blindness called LCA (Leber
congenital amaurosis), gene therapy greatly improved vision for at least a few
years. So, point 1 is correct.
o People with haemophilia are missing proteins that help their blood form clots.
Those with the most-severe forms of the disease can lose large amounts of blood
through internal bleeding or even a minor cut. In a small trial, researchers
successfully used an adeno-associated viral vector to deliver a gene for Factor IX,
the missing clotting protein, to liver cells. So, point 2 is correct
o In 2012, Glybera became the first viral gene-therapy treatment to be approved in
Europe. The treatment uses an adeno-associated virus to deliver a working copy
of the LPL (lipoprotein lipase) gene to muscle cells. The LPL gene codes for a
protein that helps break down fats in the blood, preventing fat concentrations
from rising to toxic levels. So, point 3 is correct.
o Patients with Parkinson's disease gradually lose cells in the brain that produce the
signalling molecule dopamine. As the disease advances, patients lose the ability to
control their movements. A small group of patients with advanced Parkinson's
disease were treated with a retroviral vector to introduce three genes into cells
in a small area of the brain. So, point 4 is correct.
Therefore, option (d) is the correct answer.
Knowledge Box

● There are various of types of gene therapy products, including:


o Plasmid DNA: Circular DNA molecules can be genetically engineered to carry
therapeutic genes into human cells.
o Viral vectors: Once viruses have been modified to remove their ability to cause
infectious disease, these can be used as vectors to carry therapeutic genes into
human cells.
o Bacterial vectors: Bacteria can be modified to prevent them from causing
infectious disease and then used as vectors to carry therapeutic genes into
human tissues.
o Patient-derived cellular gene therapy products: Cells are removed from the
patient, genetically modified (often using a viral vector) and then returned to
the patient.

Q33.
Answer: c
Explanation:
● India Biological Data Bank is mandated to archive all life science data generated from
publicly-funded research in India. The government has mandated that data from all
publicly funded research should be stored in this central repository. It will provide a
platform to researchers to securely store their data and provide access to a large database
of indigenous sequences for analyses. So, statement 1 is correct.
_
● The database currently stores the genomic sequences of crops such as rice, onion,
tomatoes and mustard, among others. It also stores the genomes of humans, animals,
and microbes in the same database. It will also help researchers in studying zoonotic
diseases, which spread from animals to humans. So, statement 2 is correct.
● It is supported by the Department of Biotechnology (DBT). It is established at the
Regional Centre of Biotechnology in Faridabad. It also has a data ‘Disaster Recovery’ site
at National Informatics Centre (NIC), Bhubaneshwar.
Therefore, option (c) is the correct answer.
Relevance: The ‘Indian Biological Data Bank’ has come up at the Regional Centre for
Biotechnology in Faridabad.

Q34.
Answer: a
Explanation:
● Genetic Engineering Appraisal Committee (GEAC) in India is set to recommend
‘environmental release’ of Bayer’s herbicide-tolerant cotton, Bollgard II Roundup Ready
Flex (BG-II RRF).
● The transgenic cotton — Bollgard II Roundup Ready Flex (BG-II RRF) — contains three
alien genes, the first two (‘cry1Ac’ and ‘cry2Ab’) being isolated from a soil bacterium,
Bacillus thuringiensis or Bt.
● The third gene, ‘cp4-epsps’, is sourced from another soil bacterium, Agrobacterium
tumefaciens. Its incorporation into cotton makes the crop “tolerant” to glyphosate. This
herbicide cannot be applied on normal cotton, as the chemical does not distinguish
between the crops and weeds.
Therefore, option (a) is the correct answer.
Relevance: Genetic Engineering Appraisal Committee (GEAC) is set to recommend the
‘environmental release’ of Bayer’s herbicide-tolerant cotton.

Q35.
Answer: c
Explanation:
● An exoplanet is any planet beyond our solar system. Most orbit other stars, but free-
floating exoplanets, called rogue planets, orbit the galactic center and are untethered to
any star.
● Most exoplanets are found through indirect methods: measuring the dimming of a star
that happens to have a planet pass in front of it, called the transit method. It’s pretty
rare for astronomers to see an exoplanet through their telescopes the way one might see
Saturn through a telescope from Earth. That’s called direct imaging, and only a handful
of exoplanets have been found this way.
● Exoplanets come in a wide range of sizes, from small rocky planets similar to Earth to
large gas giants similar to Jupiter. Some exoplanets are even much larger than Jupiter
and have masses several times that of Jupiter. So, a planet can exceed the size and mass
of Earth and still be categorised as an exoplanet.
_
● Exoplanets are composed of a variety of elements, and some exoplanets have been found
to have similar compositions to the planets in our solar system, such as rocky planets
with a similar composition to Earth. However, other exoplanets have been found to have
vastly different compositions, such as gas giants composed primarily of hydrogen and
helium like Jupiter and Saturn.
Therefore, option (c) is the correct answer.
Relevance: Recently, James Webb Space Telescope has discovered its first new exoplanet, LHS
475 b.

Q36.
Answer: a
Explanation:
● In the agriculture sector, nanotechnology applications can be broadly classified into the
following main sectors:
○ Crop production through the use of nano herbicides/nano pesticides and
nanomaterials to control the viral and fungal diseases of plants. So, point 1 is
correct.
o It helps in precision farming. It is an approach where inputs are utilised in precise
amounts to get increased average yields, compared to traditional cultivation
techniques. So, point 3 is correct.
○ Crop improvement using nano fertiliser and nano biosensors for soil/plant
systems, and for the nano remediation of soils, water, and the food sector.
● Although agro-nanotechnology has several benefits, nanomaterials may also be
associated with many risks, which include:
○ The degradation of the nanomaterials and/or nanocomposites under
environmental conditions, with the release of the inserted nanomaterials into the
environment.
○ A significant amount of leaching of the nanoparticles can occur in the agro-
environment, which may cause nanotoxicity. So, point 2 is not correct.
○ The integration of many nanoparticles into the human body via inhalation,
ingestion, or cutaneous contact can have many side effects on health. So, point 4
is not correct.
Therefore, option (a) is the correct answer.

Q37.
Answer: b
Explanation:
● Food irradiation is a technology that improves the safety and extends the shelf life of
foods by reducing or eliminating microorganisms and insects. Like pasteurising milk and
canning fruits and vegetables, irradiation can make food safer for the consumer. So, point
1 is correct.
● Food irradiation is used to destroy insects in or on tropical fruits and also decreases the
need for other pest-control practices that may harm the fruit. So, point 2 is correct.
_
● Food irradiation is used to inhibit sprouting and delay ripening of fruit to increase
longevity. So, point 3 is not correct.
● Food irradiation can be used to sterilise foods, which can then be stored for years
without refrigeration. Sterilised foods are useful in hospitals for patients with severely
impaired immune systems, such as patients with AIDS or undergoing chemotherapy.
Foods that are sterilised by irradiation are exposed to substantially higher levels of
treatment than those approved for general use. So, point 4 is correct.
● Food irradiation can be used for the prevention of foodborne illness effectively. It
eliminates organisms that cause foodborne illness, such as Salmonella and Escherichia
coli.
Therefore, option (b) is the correct answer.

Q38.
Answer: d
Explanation:
● Mock Meat is a plant-based product that is made from vegetarian ingredients and non-
dairy products. Soya, gluten and pea protein are some commonly used ingredients. that
are prepared to resemble the texture, consistency and mouth feel of conventional meat
and meat products. One of the main reasons why mock meat has gained popularity
among the non – vegetarians is that it simulates the aesthetic, organoleptic, and chemical
characteristics of traditional meat products. So, statements 1 and 2 are correct.
● Mock meat has healthier results. It has been claimed that they offer more nutritive value,
more fibre and lower fat content. Mock meat also does not contain cholesterol. Although
mock meat has a good source of proteins and essential amino acids, they often have
higher amounts of sodium and are highly processed. So, statement 3 is correct.
Therefore, option (d) is the correct answer.
Relevance: There is a growing demand for mock meats in India in recent times.

Q39.
Answer: c
Explanation:
● Halteria are microscopic ciliates that populate freshwater throughout the world. These
can eat huge numbers of infectious chloroviruses that share their aquatic habitat.
● These organisms can sustain themselves with viruses, consuming many of them and
growing in size. The organisms which eat viruses are called virovore.
Therefore, option (c) is the correct answer.
Relevance: Researchers have found the first known “Virovore”, an organism that eats viruses.

Q40.
Answer: d
Explanation:
● Fruit ripening is a combination of physiological, biochemical, and molecular processes
which lead to changes in colour, sugar content, acidity, texture, and aroma. It is a
physiological process that makes the fruit edible, palatable and nutritious.
_
● Ethylene gas, acetylene gas liberated from calcium carbide, and ethephon are some of
the commercial ripening agents.
● Lauryl alcohol is used as a ripening agent for bananas.
Therefore, option (d) is the correct answer.

Q41.
Answer: c
Explanation:
● The Coordinated Ocean Wave Climate Project (COWCLIP) 2.0 is an international
collaborative research project that aims to aid comprehensive assessments of climate-
driven changes in wave characteristics (ultimately of global extent), both historically and
in projected future scenarios, with the understanding of associated uncertainties.
● This dataset produced through the COWCLIP represents the first coordinated multivariate
ensemble of 21st Century global wind-wave climate projections available.
Therefore, option (c) is the correct answer.

Q42.
Answer: c
Explanation:
● Green Hydrogen is based on the generation of hydrogen (a universal, light and highly
reactive fuel) through a chemical process known as electrolysis. This method uses an
electrical current to separate the hydrogen from the oxygen in the water. So, statement
1 is correct.
● Advantages of green hydrogen:
○ 100 % sustainable: Green hydrogen does not emit polluting gases either during
combustion or during production.
○ Storable: Hydrogen is easy to store, which allows it to be used subsequently for
other purposes and at times other than immediately after its production.
○ Versatile: Green hydrogen can be transformed into electricity or synthetic gas
and used for commercial, industrial or mobility purposes. So, statement 2 is
correct.
Therefore, option (c) is the correct answer.
Relevance: The Government of India has formally approved the National Green Hydrogen
Mission, which aims to make India a global hub for the production of green hydrogen.

Q43.
Answer: c
Explanation:
● Action on Water Adaptation and Resilience (AWARe) was launched at the 27th
Conference of Parties (COP27) in 2022 of the United Nations Framework Convention on
Climate Change (UNFCCC). The initiative will have an Africa-first focus, giving hope to
drought-ravaged populations, especially in the Horn of Africa.
_
● It aims to foster political efforts in establishing pan-African water hubs. It will address
water security as part of climate change adaptation and will focus on three priorities:
○ decrease water losses worldwide and improve water supply
○ propose and support implementing policies for cooperative water-related
adaptation action and cooperation
○ interlinkages between water and climate action
● One of the critical goals of the initiative is to ensure there is 50 percent less damage from
floods and droughts by 2030.
Therefore, option (c) is the correct answer.
Relevance: Action on Water Adaptation and Resilience (AWARe) was adopted at COP-27 of
UNFCCC.

Q44.
Answer: c
Explanation:
● Black Carbon Aerosols are formed through the incomplete combustion of fossil fuels,
biofuel, and biomass. These are emitted in both anthropogenic and naturally occurring
soot. It consists of pure carbon in several linked forms. So, statement 1 is correct.
● Black carbon warms the Earth by absorbing heat in the atmosphere and by reducing
albedo, the ability to reflect sunlight when deposited on snow and ice. Black carbon
aerosols are characterised by strong light absorption. So, statement 2 is correct.
Therefore, option (c) is the correct answer.
Relevance: According to a study, since the 21st century, South Asian black carbon aerosols have
indirectly affected the mass gain of the Tibetan Plateau glaciers by changing long-range water
vapour transport from the South Asian monsoon region.

Q45.
Answer: a
Explanation:
● Naegleria is an amoeba, a single-celled organism, and only one of its species, called
Naegleria fowleri, can infect humans. It was first discovered in Australia in 1965 and is
commonly found in warm freshwater bodies, such as hot springs, rivers and lakes.
● The organism best grows in high temperatures up to 46°C and sometimes can survive at
even higher temperatures.
● Naegleria fowleri or “brain-eating amoeba” enters the human body through the nose and
then travels up to the brain. Once Naegleria fowleri goes to the brain, it destroys brain
tissues and causes a dangerous infection known as primary amoebic
meningoencephalitis (PAM).
Therefore, option (a) is the correct answer.
Relevance: Recently, South Korea reported its first case of infection from Naegleria fowleri.

Q46.
Answer: a
Explanation:
_
Sodium tetrafluoroborate (NaBF4) is a chemical compound that is used in a variety of
applications due to its unique properties. Some of the main applications and uses of sodium
tetrafluoroborate include:
● It is used as a neutralizing agent in the textile industry. In textile dyeing, printing, and
finishing processes, various chemicals are used to prepare the fabric for dyeing or
printing, and to set the dye or print. These chemicals can include acids, alkalis, and other
agents that are used to change the pH of the fabric. In this context, sodium
tetrafluoroborate is used to neutralize the excess of acid or alkali after the dyeing or
printing process. This can help to ensure that the fabric is at the correct pH for the next
step in the process and that the dye or print is properly set. So, point 1 is correct.
● It is not used as a lubricant or coolant in the automotive industry as it could cause
corrosion to the engine and other parts. So, point 2 is not correct.
● It is used as a flux in the glass industry. It helps to lower the melting point of the glass
batch, making it easier to produce glass at a lower temperature. The flux can also
improve the working properties of the glass and reduce the formation of defects during
the manufacturing process. So, point 3 is correct.
● It is not approved for use in the medical or food industries, as it is not considered safe
for consumption and has not been approved for use in products intended for human
consumption. So, point 4 is not correct.
Therefore, option (a) is the correct answer.
Knowledge Box

● NaBF4 has been used as a component in the electrolyte solutions for lithium-ion
batteries, as it helps to improve the conductivity of the electrolyte.
● Sodium tetrafluoroborate is used as a complexing agent in plating solutions and in
pickling baths to remove surface impurities.
● NaBF4 is also used as a raw material in the production of boron compounds, and as an
intermediate in the production of other chemicals and pharmaceuticals.

Relevance: Recently, Scientists from the Institute of Nano Science and Technology (INST) Mohali
have introduced a new electrolyte called Sodium tetrafluoroborate (NaBF4).

Q47.
Answer: a
Explanation:
● In India, debris avalanches and landslides occur very frequently in the Himalayas. These
also occur in Nilgiris and Western Ghats though not as frequently as in the Himalayas.
There are many factors for this:
○ Many slopes are steeper with almost vertical cliffs and escarpments in the
Western Ghats and Nilgiris. So, point 1 is correct.
○ Mechanical weathering due to temperature changes and ranges is pronounced.
So, point 3 is correct.
_
○ They receive heavy amounts of rainfall over short periods. So, there is almost
direct rock fall quite frequently in these places along with landslides and debris
avalanches. So, point 4 is correct.
● The Himalayas are tectonically active. They are mostly made up of sedimentary rocks and
unconsolidated and semi-consolidated deposits. The slopes are very steep.
● Compared to the Himalayas, the Nilgiris bordering Tamil Nadu, Karnataka, Kerala and the
Western Ghats along the west coast are relatively tectonically stable and are mostly
made up of very hard rocks. So, point 2 is not correct.
Therefore, option (a) is the correct answer.

Q48.
Answer: c
Explanation:
There are many medicinal plants found in the Himalayas, as the region is known for its diverse
and rich flora.
● Arundinella ciliata is a plant endemic to the Western Ghats. It is not a medicinal plant
and is used mostly as fodder.
● Dactylorhiza hatagirea (Salampanja) is perennial tuberous species endemic to the Hindu
Kush and Himalayan ranges of Afghanistan, Bhutan, China, India, Nepal, and Pakistan. It
is threatened by habitat loss (recently listed by IUCN as endangered), livestock grazing,
deforestation, and climate change. It is extensively used in Ayurveda, Siddha, Unani and
other alternative systems of medicine to cure dysentery, gastritis, chronic fever, cough
and stomach aches.
● Mahameda consists of dried rhizome and root of Polygonatum cirrhifolium Royle, herb
found in the temperate Himalayas. It is an Ayurvedic herb which is mentioned as one
among the Ashtavarga herbs, an Ayurvedic formulation with 8 strength promoting herbs.
Traditionally, the rhizomes of this plant are used to increase the physical strength, nourish
the body tissues, act as aphrodisiac, rejuvenative and galactagogue.
● Kshirakakoli is one among the list of endangered medicinal herbs, commonly known as
Himalayan Lilly. The ancient science of medicine, Ayurveda recommends the usage of
bulbs of Ksheerakakoli to treat general debility, rheumatism, bleeding
disorders, bronchitis and other such diseases which are mainly related to Vata and Pitta
dosha.
Therefore, option (c) is the answer.
Relevance: Three medicinal plant species found in the Himalayas have made it to IUCN Red List
of Threatened Species following a recent assessment.

Q49.
Answer: c
Explanation:
_
● Arittapatti village in Tamil
Nadu is rich in ecological
and historical significance,
it houses around 250
species of birds. It is also
home to wildlife such as
the Indian Pangolin,
Slender Loris and Pythons.
It is the first Biodiversity
Heritage Site (BHS) to be
notified in the state.
● It supports close to 72 lakes, 200 natural spring pools and three check dams. The
Anaikondan lake built during the reign of the Pandyas in the 16th century is one among
them.
● The site also features various megalithic structures, Tamil Brahmi Inscriptions, Jain Beds
and 2200-year-old rock-cut temples adding to its historical value.
Therefore, option (c) is the correct answer.
Relevance: Recently, The Tamil Nadu government issued a notification declaring the Arittapatti
village near Melur in Madurai district as a biodiversity heritage site (India’s 35th Biodiversity
Heritage Site).

Q50.
Answer: a
Explanation:
● The Man and Biosphere Program (MAB) Program is an intergovernmental scientific
programme, launched in 1971 by the United Nations Educational, Scientific and Cultural
Organization (UNESCO). It aims to establish a scientific basis for the improvement of
relationships between people and their environments.
● There is the World Network of Biosphere Reserves (WNBR) under MAB Programme. A
total of 12 biosphere reserves of India have been recognised under MAB Program.
● The Gulf of Mannar Biosphere Reserve, established in 1989, is located in Tamil Nadu. It
is the first marine biosphere reserve in India. It is part of the MAB Program. Malvathu
River of Sri Lanka and the estuaries of Thamirabarani River and Vaipar River of South India
drain into the Gulf. Marakeyars are the main tribal group in the region. So, point 1 is
correct.
● Dibru-Saikhowa is a National Park as well as a Biosphere Reserve situated in Assam. It
was designated a Biosphere Reserve in 1997. It is situated on the floodplain of
Brahmaputra. The park is bounded by the Brahmaputra and Lohit Rivers in the north and
Dibru River in the south. It is the largest salix swamp forest in north-eastern India, with
a tropical monsoon climate with a hot and wet summer and cool and usually dry winter.
It is not part of the MAB Program. So, point 2 is not correct.
● Nokrek Biosphere Reserve, established in 1988, is situated in Meghalaya. It is located in
the Northeast of India on the Tura range, which forms part of the Meghalaya Plateau.
_
Ganol, Dareng and Simsang rivers flow through the reserve. Endangered Red Panda and
Hoolock Gibbons are found in the reserve. Garo tribes are the most dominant tribe in the
area. It is part of the MAB Program. So, point 3 is correct.
● Pachmarhi Biosphere Reserve, established in 1999, is situated in Madhya Pradesh. It lies
directly at the centre of the Satpura mountain ranges. Bori Sanctuary, Satpura National
Park and Pachmarhi Sanctuary (Satpura Tiger Reserve) are protected areas within the
reserve. The Gond tribe is the most dominant tribe in the region. It is part of the MAB
Program. So, point 4 is correct.
● Kachchh Biosphere Reserve (KBR) is mainly composed of two major ecosystems called
Great Rann of Kachchh (GRK) and Little Rann of Kachchh (LRK). It covers Kachchh Desert
Sanctuary (in GRK) and Wild Ass Sanctuary (in LRK). It also covers part of the finest
grassland of Asia called “Banni”. It is not part of the MAB programme. So, point 5 is not
correct.
Therefore, option (a) is the correct answer.
Knowledge Box

Other biosphere reserves of India that have been recognized internationally under MAB
Program:
● Nilgiri (First to be included)
● Sunderban
● Nanda Devi
● Similipal
● Achanakmar - Amarkantak
● Great Nicobar
● Agasthyamala
● Khangchendzonga
● Panna

Q51.
Answer: c
Explanation:
● The Government of India Act, 1935 was passed by the British Parliament in 1935 and
came into effect in 1937.
○ It was based on a report by a Joint Select Committee, led by Lord Linlithgow, to
scrutinise the ‘White Paper’ – a scheme of constitutional proposals - prepared by
the British government close on the heels of the Round Table conferences.
● The major outcomes of the Act were:
○ The creation of a ‘Federation of India’ that consisted of two levels: a central
executive and parliament, and below it, provinces and princely states. So, point 3
is correct.
_
○ It discarded the ‘dyarchy’ system at the provincial level and allowed for the
emergence of popularly elected provincial legislatures. Dyarchy was introduced at
the centre.
○ It provided for the establishment of a Federal Court, which was set up in 1937
○ The franchise was expanded to 14% of the population from 3%.
○ Bicameral legislatures were introduced in six provinces - Bengal, Madras,
Bombay, Bihar, Assam and the United Provinces. So, point 2 is correct.
○ It abolished the Council of India, established by the Government of India Act of
1858. The secretary of state for India was provided with a team of advisors. So,
point 1 is not correct.
● Subsequent to the provisions of the Government of India Act, 1919 and the
recommendations made by the Lee Commission in 1924 for the early establishment of a
Public Service Commission, the Public Service Commission was set up in India in 1926 for
the first time. Later, the Government of India Act, 1935 envisaged a Public Service
Commission for the Federation and a Provincial Public Service Commission for each
province. So, point 4 is not correct.
Therefore, option (c) is the correct answer.

Q52.
Answer: b
Explanation:
● Under the Delhi Sultanate, Chahalgani was created by Shams-ud-din Iltutmish or
Altamash (1167-1236) after ascending to the throne. He created this group because he
realized that Turkish nobles were not trustworthy and could conspire against him to
overthrow his rule. Therefore, he had a group of faithful slaves to protect him. This group
was known as Amir-i-Chahalgani or The Forty.
Therefore, option (b) is the correct answer.

Q53.
Answer: b
Explanation:
● The "All India Harijan Sevak Sangh'' was an independent national-level non-profit
voluntary organisation. It was founded in 1932 in the wake of Mahatma Gandhi’s fast at
Yervada Jail, Pune, resulting in the historic Poona Pact. At the Second Round Table
Conference in London in 1931, Gandhi opposed the segregation of the depressed classes
of the Hindu community into a separate electoral group. Dr. Ambedkar was not
associated with the Harijan Sevak Sangh. So, point 1 is not correct.
● In 1944, the Working Committee of the All India Scheduled Castes Federation (SCF)
passed a series of resolutions outlining the safeguards for the untouchables in the new
Constitution. Dr. B.R. Ambedkar founded the Federation and organised these resolutions
as an appendix to his book ‘What the Congress and Gandhi have done to the
Untouchables’ published in 1945. So, point 2 is correct.
_
● In 1936, Dr. B.R. Ambedkar established the Independent Labour Party. The main
objective of the party was the welfare of the labourers. In order to bring the depressed
classes with the purview of the labour class, they proposed that they be referred to as the
labour class instead of the Depressed Class. So, point 3 is correct.
Therefore, option (b) is the correct answer.

Q54.
Answer: b
Explanation:
Mountbatten Plan, June 3, 1947:
● Lord Mountbatten, India's last Viceroy, released a plan on the country's independence
from the British on June 3, 1947. The important points of the plan were as follows:
○ Punjab and Bengal Legislative Assemblies would meet in two groups, Hindus and
Muslims, to vote for partition. If a simple majority of either group voted for
partition, then these provinces would be partitioned.
○ In the case of partition, two Dominions and two Constituent Assemblies would
be created. So, statement 1 is correct.
○ Sindh would take its own decision.
○ Referendums in North-West Frontier Province (NWFP) and Sylhet district of
Bengal would decide the fate of these areas. So, statement 2 is correct.
● Under the Plan, independence for princely states was ruled out and it was decided that
they would join either India or Pakistan. So, statement 3 is not correct.
Therefore, option (b) is the correct answer.

Q55.
Answer: b
Explanation:
● Sammed Shikharji is located on Parasnath hill in Jharkhand. It is considered to be the
biggest pilgrimage site by both the Digambaras and the Svetambaras subsects of Jainism.
It has been named Parasnath after the 23rd Jain Tirthankara. So, point 1 is correct.
○ Out of 24 Tirthankaras of Jains, 20 got Nirvana on Parasnath Hills.
○ A Jain temple is believed to have been constructed on the mountain by Magadha
King Bimbisara (543-491 BCE.)
● Sammed Shikharji is ranked with Ashtapad (Badrinath, Uttarakhand), Girnar (Junagadh,
Gujarat), Dilwara (Mount Abu, Rajasthan) and Shatrunjaya (Bhavnagar, Gujarat) as the
‘Svetambara Pancha Tirth’ or the five principal pilgrimage shrines. So, points 2 and 4 are
correct.
● Recently, the Madras High Court has declared that the statue of the main deity at
Thalaivetti Muniyappan temple in Salem district, Tamil Nadu is that of Buddha and not
‘Thalaivetti Muniappan’. An expert committee inspected the Thalaivetti Muniyappan
temple and concluded that the sculpture depicts several mahalakshanas (great traits) of
the Buddha. Thus, the Madras High Court ordered that the Thalaivetti Muniyappan
temple where a local deity is worshipped as per Hindu rituals, will be treated as a Buddhist
temple. So, point 3 is not correct.
_
Therefore, option (b) is the correct answer.
Relevance: Tribal bodies stepped up their demand to “free” the Parasnath hills in Jharkhand’s
Giridih district from the “clutches” of the Jain community.

Q56.
Answer: b
Explanation:
● Abdur Razzak, the Persian Islamic scholar and a Timurid chronicler visited the
Vijaynagara Kingdom at the time of Dev Raya II ruler of the Sangama dynasty. According
to Razzak’s account, Deva Raya II’s empire extended from Ceylon (now Sri Lanka) to
Gulbarga, and Orissa to Malabar. During his visit to India, he travelled extensively
throughout the Bahmani kingdom, and also visited other regions of south India as well.
He did not visit Pala Kingdom. So, pair 1 is not correctly matched.
● Ibn Battuta came to India in AD 1333 through the high mountains of Afghanistan (Hindu
Kush Mountains) and the Indus river and made his way to Delhi at the time when the
Tughlaq dynasty was at its height. Ibn Battuta reached the royal court of Mohammad
Bin Tughlaq, the then ruler of Delhi, presented him with precious gifts and in return
Battuta was given a furnished house, a job of Qadi (Judge) in the royal court by the king
and stayed for around seven years. So, pair 2 is correctly matched.
● Marco Polo was a merchant from Venice and an adventurer who travelled from Europe
to Asia from 1271 to 1295. He entered the kingdom of the Tamil Pandyas near Tanjore.
He visited Southern India during the reign of Rudramma Devi of the Kakatiya Dynasty.
During his visit to India, he accounted that it was safe to travel at night with the valuables.
The temple monasteries have both male and female deities and dark skin was highly
esteemed among them. So, pair 3 is correctly matched.
● Niccolo Conti was an Italian traveller and merchant who visited India in 1420 AD at the
Vijayanagara empire after the accession of Deva Raya II. Vijayanagara was the capital of
the Deccan of the time in India. The stone-built dwellings of the nobles, merchants, and
upper classes of the vast community elaborately constructed water-channels of the
Rayas for land irrigation. Niccolo gave a detailed account of Maha Navami festival
celebrated at the capital. So, pair 4 is not correctly matched.
Therefore, option (b) is the correct answer.

Q57.
Answer: c
Explanation:
The Turkish conquest of North India refers to the series of invasions and settlements by Turkish
and Turkic-origin Muslim dynasties in the northern regions of the Indian subcontinent. The
invasions began in the 11th century and continued until the establishment of the Mughal Empire
in the 16th century.
_
● The first wave of Turkish invaders in India came in the form of Ghaznavids, which was
long before the reign of Iltutmish. It was led by the Turkish general Mahmud of Ghazni.
He was followed by the Ghurids and the Khilji dynasty. The second wave of Turkish
invasions in India was led by the Sultanate of Delhi, established by Qutb al-Din Aibak, a
slave of the ruler of Ghur. His dynasty, known as the "Slave Dynasty" was the first Turkish
Muslim dynasty to rule in India. Under the slave dynasty, several Turkish and Turkish-
origin Muslim dynasties took control of north India and established an Islamic state. So,
statement 1 is not correct.
● Mahmud of Ghazni, the famous Turkish invader and king of the Ghaznavid Empire,
conducted a series of raids and invasions into the Indian subcontinent during the reign of
the Hindu Rajput and other regional kingdoms in the early 11th century. He is particularly
known for his raids on the wealthy temple at Somnath, which was a major center of
Hindu worship at the time. During his invasions, he defeated the Hindu rulers of the Shahi
Kingdom of Gandhara, the Rajputs, the Chalukya dynasty of Gujarat and the Paramara
dynasty of Malwa. So, statement 2 is correct.
● The Battle of Tarain was fought between Muizzuddin Muhammad and Prithviraj
Chauhan. The conflict started with rival claims for Tabarhinda. In the battle which was
fought at Tarain in 1191, the Ghurid forces were completely routed. Prithviraj later
conquered Tabarhinda after a twelve-month siege. However, little attempt was made by
Prithviraj to oust the Ghurids from Punjab. So, statement 3 is correct.
Therefore, option (c) is the correct answer.

Q58.
Answer: d
Explanation:
● The Santhals were a tribal community inhabiting large parts of Jharkhand, West Bengal,
Bihar and Odisha. The Santhals had begun to come into Bengal around the 1780s.
Zamindars hired them to reclaim land and expand cultivation, and British officials invited
them to settle in the Jangal Mahals.
● The Santhals were given a large area of land and the British persuaded them to settle in
the foothills of Rajmahal. By 1832, a large area of land was demarcated as Damin-i-Koh.
and was declared to be the land of the Santhals. They were to live within it, practise
plough agriculture, and become settled peasants. The land grant to the Santhals
stipulated that at least one-tenth of the area was to be cleared and cultivated within the
first ten years.
Therefore, option (d) is the correct answer.

Q59.
Answer: d
Explanation:
● On his resignation from the Congress presidency, Subhash Chandra Bose formed
‘Forward Bloc’ in 1939 as a radical faction within the framework of the Congress. The
_
objective behind the formation of the new party was to rally all radical and anti-
imperialist progressive elements in the country.
● In 1940, Subhash Chandra Bose convened an ‘Anti-Compromise Conference’ at
Ramgarh in Bihar under the joint auspices of the Forward Bloc and Kisan Sabha. Thus,
it was not convened by Muslim League. So, statement 1 is not correct.
● In the Anti-Compromise Conference, resolution was made to stubbornly oppose the
exploitation of Indian resources for preserving the interests of the British Empire. It
was decided in the conference that a worldwide struggle should be initiated in April
1940 to urge the people not to cooperate with the British during World War II either
with men, money or materials. So, statement 2 is not correct.
Therefore, option (d) is the correct answer.

Q60.
Answer: c
Explanation:
● Savitribai Phule was born on January 3, 1831, in Maharashtra’s Naigaon village. She was
a dalit woman from the Mali community. Savitribai, along with her husband Jyotirao
(Jyotiba) Phule, helped pioneer the education of women and backward caste
communities.
● Savitribai Phule is formally recognised as India’s first woman teacher. She established a
Mahila Seva Mandal, to fight for women’s rights where members from all castes had to
sit on the same mat — defying the prevalent casteist norms. She, along with her husband:
○ opened the first school for girls in India in 1848
○ established two educational trusts - the Native Female School, Pune, and the
Society for Promoting the Education of Mahars, Mangs and others
○ founded the Satyashodhak Samaj for equal rights, weddings, debates, discussions
and reforms
● Literary works and Poems written by Savitribai Phule:
○ 'Go, get education.'
○ 'Rise, to learn and act.'
○ 'Farming is Divine.'
○ 'Should they be called Humans?'
○ Bavan Kashi Subodh Ratnakar
Therefore option (c) is the correct answer.
Relevance: Recently, the 192nd birth anniversary of Savitribai Phule was observed.

Q61.
Answer: b
Explanation:
● Interest rates and foreign exchange rates in an economy are closely related because they
both affect the demand for a currency.
● When a country's interest rates are high, demand for its currency tends to increase
because investors are attracted to the higher returns on their investments. This can lead
to an appreciation of the currency in the foreign exchange market. Higher interest rates
_
offer lenders in an economy a higher return relative to other countries. Therefore, higher
interest rates attract foreign capital and cause the exchange rate to rise. So, statement
1 is not correct.
● Some of the main factors that can influence the relationship between interest rates and
foreign exchange rates are:
○ Trade balance of the country: A country with a trade surplus (exports exceed
imports) may see an appreciation of its currency. This is because if a country
exports more than it imports, there is a high demand for its goods, and thus, for
its currency. The economics of supply and demand dictate that when demand is
high, prices rise and the currency appreciates in value. Conversely, a country with
a trade deficit (imports exceed exports) may experience a depreciation of its
currency. This is because if a country imports more than it exports, there is
relatively less demand for its currency, so prices should decline. So, statement 2
is correct.
○ Inflation rate: High inflation can lead to a depreciation of a country's currency, as
it reduces the purchasing power of the currency. On the other hand, low inflation
may lead to an appreciation of the currency.
○ Economic growth: Strong economic growth, as evidenced from high output and
high employment, can lead to an appreciation of a country's currency, as it may
attract foreign investment and increase demand for the currency. Conversely,
weak economic growth may lead to a depreciation of the currency.
Therefore, option (b) is the correct answer.

Q62.
Answer: b
Explanation:
The Price Support Scheme (PSS) is a scheme in India that aims to protect farmers from sudden
fluctuations in the prices of certain agricultural commodities. Under the PSS, the government
purchases certain crops at a Minimum Support Price (MSP) that is announced before the sowing
season.
● Commission for Agricultural Costs and Prices (CACP) recommends MSP for twenty-two
(22) crops and Fair & Remunerative Price (FRP) for sugarcane. Apart from Sugarcane for
which FRP is declared by the Department of Food &Public Distribution, twenty-two crops
covered under MSP are Paddy, Jowar, Bajra, Maize, Ragi, Arhar, Moong, Urad,
Groundnut-in-shell, Soyabean, Sunflower seed, Sesamum, Niger seed, Cotton, Wheat,
Barley, Gram, Masur (lentil), Rapeseed/Mustard seed, Safflower, Jute and Copra. In
addition, MSP for Toria and De-Husked coconut is fixed by the Department on the basis
of MSPs of Rapeseed/Mustard seed and Copra respectively. Ginger and coffee are not
covered under the Price Support Scheme (PSS) in India.
Therefore, option (b) is the correct answer.

Q63.
Answer: d
_
Explanation:
● The money supply is all the currency and other liquid instruments in a country's economy
on the date measured. The money supply roughly includes both cash and deposits that
can be used almost as easily as cash.
● The reserve requirement is the percentage of deposits that banks are required to hold in
reserve and not lend out. If the central bank lowers the reserve requirement, it allows
banks to lend out a greater proportion of their deposits, which increases the money
supply in the economy. On the other hand, an increase in reserve requirements leads to
a decrease in the money supply in an economy. So, point 1 is not correct.
● One of the ways to increase the money supply is for the central bank/Government to
simply print more physical currency. This can be done if there is a demand for more cash
in circulation, or if the central bank wants to stimulate economic activity by increasing the
amount of money available for spending. So, point 2 is correct.
● Quantitative easing is a monetary policy tool used by central banks to increase the
money supply in the economy. It involves the central bank buying assets, such as
government securities or mortgage-backed securities, from banks, which increases the
banks' reserves and allows them to lend more, thereby increasing the money supply. So,
point 3 is correct.
● The deposit of currency into commercial banks increases the banks' deposits, which they
can then use to lend to other customers or invest in financial assets. As the banks lend
out these additional funds or invest them in financial assets, they create new money in
the form of loans or securities. This new money then enters circulation and increases the
overall money supply. So, point 4 is correct.
Therefore, option (d) is the correct answer.

Q64.
Answer: b
Explanation:
● National Company Law Appellate Tribunal (NCLAT) is a quasi-judicial body in India that
adjudicates issues relating to Indian Companies. It was constituted under the Companies
Act, 2013 for hearing appeals against the orders of the National Company Law Tribunal(s)
(NCLT). So, statement 1 is correct.
● NCLAT is the Appellate Tribunal to hear and dispose of appeals against any direction
issued or decision made or order passed by the Competition Commission of India (CCI).
So, statement 2 is correct.
○ It is also the Appellate Tribunal for hearing appeals against the orders passed by
NCLT(s) under the Insolvency and Bankruptcy Code, 2016 (IBC).
○ NCLAT is also the Appellate Tribunal to hear and dispose of appeals against the
orders of the National Financial Reporting Authority.
● No appeal shall lie to the National Company Law Appellate Tribunal from an order made
by the Tribunal with the consent of the parties. An order of the NCLAT is subject to an
appeal on a question of law to the Supreme Court. So, statement 3 is not correct.
Therefore, option (b) is the correct answer.
_
Q65.
Answer: d
Explanation:
● A Convertible Note is an instrument typically used by companies at the pre-IPO (Initial
Public Offerings) stage. It is the instrument through which registered start-ups can raise
money from investors without the requirement of a valuation report. They can be a
useful tool for companies looking to raise capital without diluting the equity of existing
shareholders too much. So, statement 1 is correct.
● It can be risky for investors, as the value of the conversion may be uncertain and may
depend on the future performance of the company. Further, investors do not have any
control or voting power when they subscribe to convertible notes. A Convertible Note has
to be repaid or converted into equity shares of a start-up company within 5 years from
the date of issuance of the Convertible Note. Since the maturity period is less, the risk
involved becomes greater. So, statement 2 is correct.
● It is issued by Private Limited Companies registered under Startup India scheme. It can
be issued only to foreign investors; Non-Resident Indian; Resident Indians; Indian
companies and Indian investors. So, statement 3 is correct.
Therefore, option (d) is the correct answer.
Knowledge Box
Other Features of Convertible Notes:
● The primary advantage of issuing convertible notes is that shares valuation is not
required. Thus, it makes it very easy for startups to raise funds without having to
worry about valuation.
● The notes can be converted into equity shares at the valuation on the date of
conversion or based on the predefined formula for calculating the value of the
company.
● They are optionally convertible whereas Compulsorily Convertible debentures are
mandatorily convertible. For investors Convertible Notes is better as it is an option
to convert it into equity.
● In India, they are regulated by the Reserve Bank of India.
Relevance: Recently, B2B e-commerce platform Udaan has raised $200 million via convertible
notes.

Q66.
Answer: c
Explanation:
● The Trade and Technology Council (TTC) is a transatlantic political body which serves as
a diplomatic forum to coordinate technology and trade policy between the United States
(US) and European Union(EU). The early agenda focused primarily on US-EU cooperation
in technology, strategic sectors, market access, trade, democratic values and rule of law
in the digital world, supply chain resilience, and the global trade order. India is not a
member of this organization. So, point 1 is not correct.
_
● In 2021, the US proposed the formation of the Chip 4 Alliance, a strategic alliance that
includes South Korea, Japan, and Taiwan. Overall, the intention of the alliance is to
restructure the global supply chain such that it is less reliant on China, with a particular
focus on the following goals: Geographically diversify manufacturing capacity away from
China; Protect the intellectual property of companies from member nations; Coordinate
uniform export controls with regard to China; Encourage favourable distribution terms
among friendly nations. India is not a member of this group. So, point 2 is not correct.
● The Quad Semiconductor Supply Chain Initiative is a partnership between the
governments of the United States, Japan, Australia, and India to promote the
development of a resilient and secure semiconductor supply chain in the Indo-Pacific
region. So, point 3 is correct.
● The World Semiconductor Council (WSC) is an international forum that brings together
industry leaders to address issues of global concern to the semiconductor industry. It is
comprised of the Semiconductor Industry Associations (SIAs) of the United States, Korea,
Japan, Europe, China and Chinese Taipei. Indian semiconductor industry associations are
not members of this organization. So, point 4 is not correct.
Therefore, option (c) is the correct answer.
Relevance: There is an increasing concern about the security of the semiconductor supply chain
across the world.

Q67.
Answer: c
Explanation:
● Digital Agristack is a collection of technologies and digital databases that focuses on
farmers and the agricultural sector. AgriStack will create a unified platform for farmers
to provide them end-to-end services across the agriculture food value chain. It will help
the government in effective planning towards increasing the income of farmers in
particular and improving the efficiency of the agriculture sector as a whole. So, statement
1 is correct.
● Under it, each farmer will have a unique digital identification that contains personal
details, information about the land they farm, as well as production and financial details.
Each ID will be linked to the individual's digital national ID Aadhaar. So, statement 2 is
correct.
● Digital Agristack is a federated structure and ownership of data is with the States only.
Access to federated farmers’ databases is with the Government only. No private
company is involved in developing the federated farmers’ database. So, statement 3 is
not correct.
Therefore, option (c) is the correct answer.
Relevance: Recently, the Ministry of Agriculture & Farmers Welfare has commenced the work
for creating Digital Agristack in the country.

Q68.
_
Answer: c
Explanation:
● In India, the Securities and Exchange Board of India (SEBI) regulates spot markets and
derivative markets. SEBI is the regulator for the securities market in India and is
responsible for protecting the interests of investors, promoting the development of the
securities market, and regulating the securities market.
● A spot market is a financial market in which financial instruments, such as currencies,
commodities, and securities, are bought and sold for immediate delivery. In this,
transactions are settled on the spot, which means that the buyer pays the seller the
agreed-upon price for the asset at the time of the transaction.
● A derivative market is a financial market that trades derivatives, which are financial
instruments that are derived from other assets.
○ Derivatives are contracts between two parties that derive their value from an
underlying asset, such as a currency, commodity, or security.
○ The underlying asset can be any asset, such as a physical commodity, a financial
instrument, or an index.
Therefore, option (c) is the correct answer.
Knowledge Box
Securities and Exchange Board of India (SEBI)
● It is an independent body established in 1992 through the Securities and Exchange
Board of India Act, 1992.
● It operates under the jurisdiction of the Ministry of Finance but is an autonomous
body that operates independently of the government.
● It is headquartered in Mumbai.
● Its main functions include:
○ Issuing licences to stock exchanges and intermediaries such as brokers,
merchant bankers, and mutual funds
○ Setting standards for the operation of stock exchanges and intermediaries
○ Monitoring the securities market to ensure compliance with regulations
○ Investigating and enforcing action in cases of violations of securities laws
and regulations
○ Promoting investor education and awareness
○ Regulating the issue and listing of securities
○ Regulating the trading of securities, including the operation of spot and
derivative markets
Relevance: Recently, the Securities and Exchange Board of India (SEBI) has extended the ban on
futures trading in seven Agri commodities.

Q69.
Answer: b
Explanation:
_
● Infrastructure as a Service is a form of cloud computing that delivers fundamental
computing, network, and storage resources to consumers on-demand, over the internet,
and on a pay-as-you-go basis. It enables end users to scale and shrink resources as per
the requirement by reducing the need for high, up-front capital expenditures or
unnecessary “owned” infrastructure. It emerged as a popular computing model in the
early 2010s, and since that time, it has become the standard abstraction model for many
types of workloads.
● Software as a service is a cloud-based software delivery model in which the cloud
provider develops and maintains cloud application software which provides automatic
software updates and makes software available to its customers via the internet on a
pay-as-you-go basis. The first generation of Software as service solutions was evolved,
inflexible, and designed to solve a single business problem in the early 2000.
● Platform-as-a-Service is a cloud computing model that provides customers a complete
cloud platform of hardware, software, and infrastructure for developing, running, and
managing applications without the cost, complexity, and inflexibility that often comes
with building and maintaining that platform on-premises.
● Desktop as a service is a cloud computing offering in which a third party hosts the back
end of a virtual desktop infrastructure (VDI) deployment. In Desktop as a service,
desktop operating systems run inside virtual machines on servers in a cloud provider's
data centre. The entire necessary support infrastructure, including storage and network
resources, also lives in the cloud. Desktop as a service provider streams virtual desktops
over a network to a customer's endpoint devices, where end users may access them
through client software or a web browser.
Therefore, option (b) is the correct answer.

Q70.
Answer: d
Explanation:
● The Harappan civilization, also known as the Indus Valley Civilization (IVC), was a Bronze
Age civilization that flourished in the Indus Valley in the northwest Indian Sub-continent.
● The civilization was known for its sophisticated urban planning, and its cities were
characterized by well-planned streets, buildings, and public spaces. Some of the most
notable Harappan towns include Harappa, Mohenjo-Daro, Chanhudaro, Kalibangan and
Lothal.

Sl. Harrapan Features and Significant Discoveries


No. Towns
1. Chanhudaro ● It has provided evidence of factories producing a variety of
figurines, seals, toys, and bone instruments, suggesting that
it was a village with a large number of craftsmen and an
industrial town.
_
● Sesame, a South African native, has been found in a number
of Harappan sites, including Chanhudaro, where it was
likely planted for oil.
● It is the only Harappan city without a walled citadel. So,
pair 1 is correctly matched.
● An impressive factory known as the Bead Making Factory
was discovered, complete with a furnace.
● Large number of copper tools have been found
2. Kalibangan ● A wooden furrow and seven fire altars in a row have been
discovered, implying the practice of sacrifice. So, pair 2 is
correctly matched.
● Camel bones have been discovered here.
● No drainage system.
● It has provided evidence of the first recorded earthquake.
The earthquake, which occurred around 2600 BC, is said to
have led to the demise of the Indus Valley Civilization.
3. Lothal ● Lothal copper is very clean, missing the arsenic often
utilised by coppersmiths throughout the Indus valley.
● There is existence of private and public fire-altars
● Its most notable feature is its artificial dockyard, which
makes it a major location for international archaeology.
● Evidence of Mix Cropping is found at Kalibangan and not at
Lothal. So, pair 3 is not correctly matched.
3. Rakhigarhi ● Largest IVC site in India (also claimed to be largest IVC site)
● Painted potteries in large numbers (show rich and
dominant people lived)
● DNA finding of Skeletal remains of a woman
● Manufacturing center of terracotta (Idly shaped
terracotta). So, pair 4 is correctly matched.
Therefore, option (d) is the correct answer.

Q71.
Answer: d
Explanation:
● Article 131 - Original Jurisdiction of the Supreme Court: The Supreme Court, to the
exclusion of any other court, has original jurisdiction in any dispute between different
units of the Indian Federation. It decides on any dispute:
○ Between the Centre and one or more states;
○ Between the Centre and any state or states on one side and one or more other
states on the other side;
○ Between two or more states.
● In the above federal disputes, the Supreme Court has exclusive original jurisdiction. It
means, no other court can decide such disputes and original means, the power to hear
such disputes in the first instance, not by way of appeal.
_
Therefore, option (d) is the correct answer.
Relevance: Both Maharashtra and Karnataka states passed a unanimous resolution to support a
legal battle to resolve the boundary dispute.

Q72.
Answer: d
Explanation:
● The Department for Promotion of Industry and Internal Trade (DPIIT) is a government
agency in India that is responsible for promoting and developing industries within the
country. DPIIT was formerly known as the Department of Industrial Policy and Promotion
(DIPP) before it got reorganized in 2019. DPIIT is under the Ministry of Commerce and
Industry and works closely with other government agencies and organizations to achieve
its objectives. Some of the functions of DPIIT include:
● DPIIT manages and promotes Intellectual Property Rights (IPR) in the country. The
following legislations are administered by the Department: The Industries (Development
and Regulation) Act, 1951; The Explosives Act, 1884; The Inflammable Substances Act,
1952; The Boilers Act, 1923; The Copyright Act, 1957; The Patents Act, 1970; The Design
Act, 2000; The Geographical Indications of Goods (Registration and Protection) Act,
1999; The Trade Marks Act, 1999; The National Institute of Design Act, 2014. So,
statement 1 is correct.
● The Department is the nodal department for the formulation of the policy of the
Government on Foreign Direct Investment (FDI) and handles matters related to FDI and
undertakes the promotion of investment for the industrial development of the country.
There are five territorial divisions for international cooperation and industrial promotion
handling matters emanating from the Americas, Europe, Africa and the Middle East, and
Asia and Oceania. So, statement 2 is correct.
● DPIIT is the nodal department for the promotion of productivity and quality in the
industrial sector. Under it, the National Productivity Council (NPC) represents India in the
Tokyo-based Asian Productivity Organization (APO), of which the Government of India is
a founder member and implements APO programmes/activities relating to India. NPC
undertakes productivity augmentation through domain-specific consultancy, training,
workshops and seminars. So, statement 3 is correct.
Therefore, option (d) is the correct answer.

Q73.
Answer: a
Explanation:
● The Fifth Schedule of the Constitution of India designates tribal majority areas in ten tribal
minority states within peninsular India including, Andhra Pradesh, Telangana, Gujarat,
Jharkhand, Chhattisgarh, Himachal Pradesh, Madhya Pradesh, Maharashtra, Odisha, and
Rajasthan.
_
● The President is empowered to declare an area to be a scheduled area. He can also
increase or decrease its area, alter its boundary lines.
● The executive power of a State extends to the scheduled areas therein. But the Governor
has a special responsibility regarding such areas. S/He has to submit a report to the
President regarding the administration of such areas, annually or whenever so required
by the President.
● Each State having scheduled areas has to establish a Tribes Advisory Council to advise on
the welfare and advancement of the Scheduled Tribes. It is to consist of 20 members,
three-fourths of whom are to be the representatives of the Scheduled Tribes in the State
Legislative Assembly. The Governor is empowered to direct that any particular act of
Parliament or the State Legislature does not apply to a Scheduled Area or apply with
specified modifications and exceptions.
● It is under the Sixth Schedule that the tribal areas in the four states of Assam, Meghalaya,
Tripura and Mizoram can constitute Autonomous Districts Councils.
Therefore, option (a) is the correct answer.

Q74.
Answer: c
Explanation:
● In Justice K.S. Puttaswamy (Retd.) and Anr v. Union of India and Ors Case (2017), the
decision of a woman to procreate or abstain from procreating has been recognized as a
facet of her right to lead a life with dignity and the right to privacy under Article 21 of
the Constitution.
● In Suchita Srivastava v Chandigarh Administration case (2009), Supreme Court
recognized that a woman’s right to reproductive autonomy is a dimension of Article 21
of the Constitution. The Supreme Court said that a woman’s right to reproductive choice
is an inseparable part of her personal liberty under Article 21 of the Constitution of India.
She has a sacrosanct right to bodily integrity.
● The judgement held that reproductive rights include a woman’s entitlement to carry a
pregnancy to its full term, to give birth, and to subsequently raise children; and that these
rights form part of a woman’s right to privacy, dignity, and bodily integrity.
Therefore, option (c) is the correct answer.

Q75.
Answer: b
Explanation:
● The Wildlife (Protection) Act of 1972 is a law in India that provides for the protection of
wild animals and plants, and aims to conserve their ecosystems. The Act established
several institutions and authorities to help implement and enforce the law, some of which
include:
● National Board for Wildlife (NBWL) is a statutory body established by the Wildlife
(Protection) Act of 1972, which is responsible for advising the government on matters
_
related to wildlife conservation and management, and for approving development
projects in and around protected areas. So, point 1 is correct.
● The National Biodiversity Authority (NBA) is an autonomous statutory body established
under the Biological Diversity Act, 2002 (BDA). The NBA is responsible for implementing
the provisions of the Act, which aims to conserve and sustainably use the country's
biodiversity. So, point 2 is not correct.
● Central Zoo Authority is established under the Wildlife (Protection) Act of 1972 to
complement and strengthen the national effort in the conservation of the rich
biodiversity of the country. So, point 3 is correct.
● The National Tiger Conservation Authority (NTCA) has been constituted under the
Wildlife (Protection) Act, 1972. It manages the Project Tiger which is launched to protect
the Bengal tiger and its habitat. So, point 4 is correct.
Therefore, option (b) is the correct answer.
Knowledge Box
Institutions/Authorities created under Wildlife Protection Act 1972:
● National Parks and Sanctuaries: Under the Wildlife Protection Act 1972, the
government may declare any area to be a National Park, Sanctuary, Conservation
Reserve or Community Reserve for the protection, propagation or development of
wildlife or its environment.
● State Board for Wildlife (SBWL): The Act also established SBWL for each state, which
are responsible for advising the state governments on matters related to wildlife
conservation and management, and for approving development projects in and around
protected areas.

Q76.
Answer: d
Explanation:
● The United Nations Convention on Biological Diversity (CBD), informally known as the
Biodiversity Convention, is a multilateral treaty opened for signature at the Earth Summit
in Rio De Janeiro in 1992. The three major climate conventions that were formed in the
Rio Earth Summit 1992 are a) The UN Framework Convention on Climate Change
(UNFCCC), b) The Convention on Biological Diversity (CBD) and c) the Convention to
Combat Desertification (CCD). So, statement 1 is correct.
● It meets every two years to work on a global plan to halt biodiversity loss and restore
natural ecosystems. The CBD’s governing body is the Conference of the Parties (COP). This
ultimate authority of all governments (or Parties) that have ratified the treaty meets
every two years to review progress, set priorities and commit to work plans. The
Secretariat of the Convention on Biological Diversity (SCBD) is based in Montreal, Canada.
● The Cartagena Protocol on Biosafety to the CBD is an international agreement which aims
to ensure the safe handling, transport and use of living modified organisms (LMOs)
_
resulting from modern biotechnology that may have adverse effects on biological
diversity, taking also into account risks to human health. It was adopted on 29 January
2000 and entered into force on 11 September 2003. So, statement 2 is correct.
● The CBD is the international legal instrument for "the conservation of biological diversity,
the sustainable use of its components and the fair and equitable sharing of the benefits
arising out of the utilisation of genetic resources" that has been ratified by 196 nations.
The convention is legally binding on its signatories. So, statement 3 is correct.
Therefore, option (d) is the correct answer.
Knowledge Box
The 2022 United Nations Biodiversity Conference (COP15) of the Parties to the UN Convention
on Biological Diversity (CBD) was concluded in Montreal, Canada recently. The Montreal
Conference has delivered a new agreement called the Global Biodiversity Framework (GBF),
which contains four goals and 23 targets that need to be achieved by 2030. The following are
the major targets set under GBF:
● 30x30 Target:
o Restore 30% degraded ecosystems globally (on land and sea) by 2030
o Conserve and manage 30% areas (terrestrial, inland water, and coastal and marine)
by 2030.
● Stop the extinction of known species, and by 2050 reduce tenfold the extinction risk
and rate of all species (including unknown)
● Reduce risk from pesticides by at least 50% by 2030
● Reduce nutrients lost to the environment by at least 50% by 2030
Relevance: Recently, the Convention on Biological Diversity (CBD) was concluded in Montreal,
Canada.

Q77.
Answer: b
Explanation:
● The Moscow Format is one of the several dialogue platforms on Afghanistan — which
began before the Taliban takeover of Kabul. It consists of Russia, China, Pakistan, Iran,
Kazakhstan, Tajikistan, Kyrgyzstan, Uzbekistan, Turkmenistan and India.
● The key objective of the Moscow format of consultations is to facilitate the national
reconciliation process in Afghanistan to establish peace. It is often used as an alternative
to formal peace talks, and can involve a range of stakeholders, including governments,
international organisations, civil society groups, and other non-governmental
organisations.
Therefore, option (b) is the correct answer.
Relevance: Recently, India joined the latest meeting of the ‘Moscow format consultations on
Afghanistan’.

Q78.
Answer: d
_
Explanation:
● The United Nations Country Team (UNCT) includes all the UN entities working on
sustainable development, emergency, recovery and transition in programme countries.
So, statement 1 is correct.
● The UNCT is led by the UN Resident Coordinator, who is the representative of the UN
Secretary-General in a given country. The Resident Coordinator (RC) is the highest
ranking representative of the United Nations development system (UNDS) at the country
level with the responsibility to lead United Nations country teams. So, statement 2 is
correct.
● The UNCT exists in 132 countries, covering all of the 162 countries where there are United
Nations programmes.
● The UN Sustainable Development Group oversees the coordination of development of
the UNCT in 162 countries and territories. It also guides, supports and tracks the
development of programmes. So, statement 3 is correct.
Therefore, option (d) is the correct answer.
Relevance: Union Minister of Environment Forest and Climate Change addressed the special
meeting of the UN Country Team (UNCT) on UNFCCC CoP 27.

Q79.
Answer: b
Explanation:
● The University Grants Commission (UGC) is a statutory organisation in India that was
established in 1956 by the Indian Union government. The UGC was set up based on the
recommendations of the University Education Commission (1948-1949), which was
headed by Dr. S. Radhakrishnan. So, statement 1 is not correct.
● According to the UGC Act, the Commission consists of a Chairman, a Vice-Chairman, and
ten other members and it is appointed by the Central Government. It advises the Central
and State governments on the measures necessary for the improvement of university
education.
● The main functions of the UGC are to determine and maintain standards of teaching,
examination, and research in universities; disburse grants to universities and colleges;
promote and coordinate university education; determine the recognition of universities
and colleges. UGC also provides approval to foreign universities willing to set up campuses
in India. So, statement 2 is correct.
Therefore, option (b) is the correct answer.
Relevance: Recently, the University Grants Commission has released Academic Collaboration
between Indian and Foreign Higher Education Institutions

Q80.
Answer: d
Explanation:
_
● Carbon leakage refers to the phenomenon in which companies or industries relocate to
countries with less stringent regulations on carbon emissions in order to avoid the costs
associated with reducing their own emissions.
● Carbon leakage can occur when a country or group of countries implements a carbon
pricing mechanism, such as a carbon tax or cap-and-trade system, and companies that
are affected by these regulations move their operations to countries without similar
regulations in order to avoid the costs. As a result, overall global emissions may not
decrease as much as intended because the carbon-intensive activities are still taking
place, just in a different location.
● Carbon leakage can be mitigated by implementing border carbon adjustments, to
account for the carbon embedded in the products imported to a country.
Therefore, option (d) is the correct answer.

Q81.
Answer: d
Explanation:
● The Himalayan Mountain range
in India is home to a number of
riverine landforms. These
include valleys, gorges, and
canyons that have been formed
by the erosion of rivers over
time. The main landforms can
be arranged from north to
south with respect to their
occurrence of formation:
● A glacial lake is a lake that has
been formed by the melting of
a glacier. These lakes are
typically found in areas that
were once covered by glaciers, such as mountain valleys and polar regions.
● The V-shape of the valley is formed by the erosion of the sides of the valley by the river,
usually in the highland region. As the river flows, it carries sediment and rocks with it,
which erode the sides of the valley.
● Meanders are winding patterns that are formed in rivers as they flow across the
landscape in lowland regions. These patterns are created as the river erodes the land on
the outside of a bend and deposits sediment on the inside of the bend. Over time, this
process can create a looping pattern known as a meander.
● An estuary is a body of water where a river meets the sea. It is a place of transition
between freshwater and saltwater and is characterised by the mixing of these two types
of water. Estuaries are formed in the lower region of a river where the river's flow is
slowed down by the resistance of the sea.
_
So, the order of Himalayan landforms from north to south with respect to their occurrence of
formation is - Glacial lakes - V-Shaped valleys – Meanders – Estuaries.
Therefore, option (d) is the correct answer.

Q82.
Answer: d
Explanation:
● Kuki chin communities are collectively called Zo people. The Kuki chin people are settled
in the Chittagong hill tracts, the only extensive hill area in Bangladesh that lies in the
southeastern part of the country. This community is different from the majority Muslim
population of Bangladesh. They are fleeing Bangladesh to escape the fighting between
the Bangladesh army and Kuki chin national front. So, pair 1 is correctly matched.
● Rohingya are an ethnic group that predominantly live in the Western Myanmar province
of Rakhine. They speak a dialect of Bengali, as opposed to the commonly spoken Burmese
language. The Rakhine region of Northern Myanmar is largely believed to be the original
home of the Rohingyas. So, pair 2 is correctly matched.
● The Birta and Hausa tribes live on both sides of the Sudan-Ethiopia border. The
Sudanese Hausa tribe are part of the much larger Hausa ethnic group, which is
particularly prominent in West Africa. Dozens of families are fleeing violence in Sudan’s
Blue Nile State, where there are continuing clashes between Birta and Hausa tribes. So,
pair 3 is correctly matched.
Therefore, option (d) is the correct answer.

Q83.
Answer: c
Explanation:
● Mass movements transfer the mass of rock debris down the slopes under the direct
influence of gravity. The movements of mass may range from slow to rapid, affecting
shallow to deep columns of materials and include creep, flow, slide and fall. Mass
movements are very active over weathered slopes rather than over un-weathered
materials. So, statement 1 is correct.
● Gravity exerts its force on all matter, both bedrock and products of weathering.
Weathering is not a prerequisite for mass movement though it aids mass movements.
So, statement 2 is not correct.
Statement 1 is correct but Statement 2 is not correct
Therefore, option (c) is the correct answer.

Q84.
Answer: a
Explanation:
● Sacred groves are small patches of forest or vegetation that are considered sacred and
protected by local communities. They are often associated with specific deities or
_
religious beliefs and are considered to be protected areas for both the plants and animals
that live there, as well as the spirits or deities that are said to inhabit the grove.
Sl. States Sacred Groves
No
.
1. Tamil Nadu ● "Kovil Kaadugal" is a term used in the Tamil Nadu
state of India to refer to sacred groves. It is a Tamil
word which consists of two parts, 'Kovil' means
temple and 'Kaadu' means forest or grove.
● Together they refer to the small forests or groves that
are considered sacred and protected by local
communities and it is usually found near the temples.
● These are considered as the abode of local deities and
are protected by the communities that live near them.
● They serve as important places of worship and are
often used for rituals and ceremonies and are
considered to be sacred places of pilgrimage. So, pair
1 is correctly matched.

2. Rajasthan ● The vanis of Mewar, the kenkris of Ajmer, the orans of


Jodhpur, Jaisalmer and Bikaner and the shamlat dehs
of Alwar constitute the sacred groves of Rajasthan.
● The orans, filled with khejarli trees (Prosopis
spicigera), deer, blackbuck and nilgai (Blue Bull), are
sacred to Bishnois.
Recently, 60 activists came together to create
awareness over the longest ‘Oran Bachao Yatra’ and
called for urgent action to preserve the sacred groves
as lifelines for the desert. So, pair 2 is correctly
matched.

3. Odisha ● Local term for groves in Odisha is Jahera,


Thakuramma.
● Sacred groves of Odisha harbour some unique socio-
cultural plants like Saraca asoca, Mesua ferrea,
Memecylon umbellatum, Michelia champaca,
Bombax ceiba, Murraya paniculata, Couroupita
guianensis.
● The sacred grooves in Jammu & Kashmir are known
as Banis. So, pair 3 is not correctly paired.
_
4. Karnataka ● The groves in Karnataka broadly come under two
classes: smaller groves are called Kans that are
entirely protected. Larger groves are known as
Devarkadu or Devarkan which function as resource
forests, offering both sustenance and ecological
security.
● A unique feature in these groves is the offering of
terracotta hounds in the groves of Kodagu.
● Local terms for these groves are Huli devarakadu,
Nagavan, Bhatappavana, Jatakappan bana, Ghodi
Bana, etc.
● Sarana or Jaherthan are the local terms for groves in
Jharkhand. So, pair 4 is not correctly matched.
Therefore, option (a) is the correct answer.
Relevance: According to a study there is a decrease in sacred groves area in many places due to
rampant encroachments, the spread of invasive species, and unfettered human interference.

Q85.
Answer: b
Explanation:
The Sahitya Akademi Award is an Indian literary award presented annually by the Sahitya
Akademi, India's National Academy of Letters. The Sahitya Akademi Award is a highly respected
literary award in India and winning it can be a significant achievement for an Indian writer. The
award is given in various categories, including poetry, fiction, and drama. It was founded in the
year 1954, and has been given annually since then.
● The award is given annually to those who have achieved literary merit and created new
trends by publishing their works, prose or poetry, in any of the 24 major languages
recognised by the Akademi. Besides the 22 languages enumerated in the Constitution of
India, Sahitya Akademi has recognised English and Rajasthani as languages which can be
considered for the award. So, statement 1 is not correct.
● The award is given to writers of Indian citizenship for a book of literary merit, published
during the five years prior to the year preceding the year of award. However, it is not
given to a work of translation or any anthology of multiple authors. So, statement 2 is
correct.
● Sahitya Akademi Award can be given for the posthumous publication of a book. The
award is given regardless of whether the author is alive or deceased. If an author passes
away before the book is published, it is still eligible for the award as long as it is published
within five years of their death. So, statement 3 is not correct.
Therefore, option (b) is the correct answer.
Relevance: Sahitya Akademi Awards were announced in December 2022.
_
Q86.
Answer: a
Explanation:
● Nilgiri Biosphere Reserve was the first
biosphere reserve in India established in 1986.
● It is located in the Western Ghats and falls under
the biogeographic region of the Malabar
rainforest. It encompasses parts of Kerala,
Karnataka and Tamil Nadu.
● Following are the protected areas within the
reserve:
○ Mudumalai Wildlife Sanctuary
○ Wayanad Wildlife Sanctuary
○ Bandipur National Park
○ Nagarhole National Park
○ Mukurthi National Park
○ Silent Valley National Park
● The genus Baeolepis is exclusively endemic to
the Nilgiris.
Therefore, option (a) is the correct answer.

Q87.
Answer: d
Explanation:
● Zaporizhzhia nuclear plant is the largest nuclear power plant in Europe. It is located in
southern Ukraine on the banks of the Kakhovka reservoir on the Dnipro River. It has been
in the news in the context of the Russia-Ukraine Conflict. So, pair 1 is correctly matched.
● Since the murder of President Jovenel Moïse in July 2021, Haiti has been paralysed
by political gridlock and rampant gang violence. The gangs control more than half of the
country and suffocate the capital, Port-au-Prince, by blocking roads and imposing a reign
of terror. So, pair 2 is correctly matched.
● The 15th Conference of Parties of the United Nations Convention to Combat
Desertification (UNCCD) was concluded in 2022 in Abidjan with a global pledge to boost
drought resilience and invest in land restoration for future prosperity. The UNCCD
COP15 adopted various decisions, including on tenure, migration and gender, which
highlight the role of land in addressing multiple crises. So, pair 3 is correctly matched.
Therefore, option (d) is the correct answer.
Relevance: Officials from the International Atomic Energy Agency (IAEA) assessed the condition
of the Zaporizhzhia Nuclear Plant.

Q88.
Answer: b
Explanation:
_
● The Schengen Zone is named after the 1985 Schengen Agreement, which established the
rules for passport-free travel within the zone.
● Schengen Zone is an area in Europe where borders are open and passport-free travel is
allowed. This means that citizens of Schengen member countries can travel freely
between these countries without having to go through border checks or show their
passports.
● Currently, the Schengen Area consists of 27 member countries. All of these countries are
located in Europe. Croatia is the latest country to join the Schengen Area in 2022.
● Some European Union (EU) member countries, such as the United Kingdom and Ireland,
have chosen to opt out of the Schengen Agreement and maintain their own separate
rules for border controls and immigration.
● The membership in the Schengen Zone is not limited to EU member countries. Iceland,
Norway, Switzerland and Lichtenstein are associate members of the Schengen Area but
are not members of the EU.
● The Schengen Agreement allows member countries to temporarily reintroduce border
controls in certain circumstances, such as during times of crisis or to address specific
security concerns.
Therefore, option (b) is the correct answer.
Relevance: Recently, Croatia became the 27th European country to join the Schengen area.

Q89.
Answer: c
Explanation:
● Mimicry is an adaptive phenomenon in which a palatable organism resembles an
unpalatable organism to deceive predators. The unpalatable one is called models and the
palatable one is called mimics. So, statement 1 is correct.
○ Palatable organisms are those organisms which are attractive to predators while
unpalatable are those that do not attract predators.
○ On the other hand, camouflage is the ability an organism has to be unnoticed by
its predators (or prays) by copying some environmental traits or by developing a
disruptive colouration that allows it to hide.
● Mimicry in butterflies is not limited to the resemblance in wing colour patterns alone, as
some mimics have also evolved to imitate the flight behaviours of model species. In
nature, multiple models and mimic butterflies could be found in the same habitat at the
same time. These similar-looking co-occurring butterflies together form a mimetic
community. So, statement 2 is correct.
Therefore, option (c) is the correct answer.
Relevance: A study conducted by scientists of the National Centre for Biological Sciences (NCBS)
has discovered secrets through which butterflies warn, fool, and escape their predators using
their wing colour patterns and flight behaviour.

Q90.
Answer: b
Explanation:
_
● The National Credit Framework (NCrF), is the first of its kind framework in India, which
aims at integrating academic education and skilling in India. The NCrF is a unified credit
accumulation and transfer framework applicable to school, higher and vocational
education.
● It is an umbrella framework, encompassing all the other qualification frameworks for
school, higher and vocational education (such as National School/Higher Education
Qualification Framework). This framework creditises learning i.e., assigns credit-based
value to different levels of learning- including academics, skilling and experiential
learning. Credit will be a way to recognize and quantify the learning levels and learning
outcomes at different stages of education/vocation.
● As per the NCrF, any credit will only be attributed after an assessment, the specifics of
which are to be determined by the concerned regulatory (such as UGC, AICTE, NCVET,
NCERT, NIOS etc.). NCrF is only an enabling framework and does not hold regulatory
mandates.
Therefore, option (b) is the correct answer.
Relevance: The Ministry of Education has recently launched the draft report on the National
Credit Framework.

Q91.
Answer: a
Explanation:
● The Ashtadhyayi is a Sanskrit grammatical treatise written by the ancient Indian
grammarian Pāṇini, probably around 4th century BCE. It consists of 3,959 sutras
(aphorisms) and is considered to be the earliest extant work of Indian linguistics. It is also
one of the earliest known works on descriptive linguistics, and has been extremely
influential in the development of the study of linguistics in the Indian subcontinent. So,
pair 1 is correctly matched.
● Mahabhasya is a commentary on Ashtadhyayi written by the ancient Indian Grammarian
Patanjali. It is written in the 2nd century BCE and considered as an important work in the
field of Indian linguistics. It provides an explanation of the meaning and use of each sutra
in the Ashtadhyayi, as well as insight into the grammar and structure of the Sanskrit
language. So, pair 2 is not correctly matched.
● Kasika Vritti is a commentary on the Mahabhasya by the ancient Indian grammarian
Jayaditya and Vamana. It is written in the 7th century CE and is considered as an
important source for understanding the Mahabhasya and Ashtadhyayi. It provides further
clarification and explanation of the concepts discussed in the Mahabhasya, as well as
additional examples and exercises to help students understand and apply the rules and
principles outlined in the Mahabhasya. An ancient author Kautilya wrote Arthashastra, a
treatise on statecraft, economic policy and military strategy. So, pair 3 is not correctly
matched.
Therefore, option (a) is the correct answer.
_
Q92.
Answer: d
Explanation:
● The Ahoms were a Mongoloid tribe from north Burma who had succeeded in
establishing a powerful kingdom in the thirteenth century and had become Hinduized in
course of time. The name Assam is derived from them. So, statement 1 is correct.
● Chaolung Sukapha was a 13th-century ruler who founded the Ahom kingdom that ruled
Assam for six centuries. Contemporary scholars trace his roots to Burma. Sukapha was a
leader of the Ahoms. He reached Brahmaputra valley in Assam from upper Burma in the
13th century with around 9,000 followers. So, statement 2 is correct.
○ Chaolung Sukapha subjugated the Chutias, Morans, Borahis, Nagas, Kacharis and
the Kamata kingdom (Kamrup).
○ His son Suteupha (1268-1281) further extended his domain towards the southern
banks of Brahmaputra up to Kalang (modern north-Cachar sub-division) by
defeating the Kacharis.
○ The Ahoms became a paramount power in the whole of the Brahmaputra Valley
under Sukhangpha (1293-1332).
● The British had pledged to withdraw from Assam after the First Burma War (1824-26).
But, after the war, instead of withdrawing, the British attempted to incorporate the
Ahoms’ territories in their dominion. This sparked off a rebellion in 1828 under the
leadership of Gomdhar Konwar, an Ahom prince, along with compatriots, such as
Dhanjoy Bongohain, and Jairam Khargharia Phukan. Assembling near Jorhat, the rebels
formally made Gomdhar Konwar the king. Finally, the Company decided to follow a
conciliatory policy and handed over Upper Assam to Maharaja Purandar Singh Narendra
and part of the kingdom was restored to the Assamese king. So, statement 3 is correct.
Therefore, option (d) is the correct answer.

Q93.
Answer: b
Explanation:
● Bhagabat Chatuspathi was founded by Satishchandra Mukherjee. He was an eminent
advocate of national education at Bhawanipur in 1895. The birth of the National Council
of Education was facilitated by the previous establishment of other educational
institutions in Bengal and outside.
● It was dedicated to the study of Indian religion and philosophy primarily and to Indology
in general. The objective was to satisfy the quest for national identity. Its classes were
held in Modern Vidyasagar College in the evening. At the same time, the famous Dawn
Magazine was also launched. Many articles on Indology appeared in its pages till 1902
when it became the mouthpiece of the Dawn society.
Therefore, option (b) is the correct answer.

Q94.
Answer: c
Explanation:
_
● Mughal Emperor Akbar (1556-1605) ordered several translations of Sanskrit texts like
Ramayana and several Upanishads into Persian. The translation was made by a scholar
named Mulla Daud. The translation was completed in 1585 and was titled "Gulshan-i-
Raz," which means "The Rose Garden of Secrets."
● His great-grandson Dara Shikoh continued the efforts. Dara translated the Upanishads
and the Bhagavad-Gita into Persian.
● Akbar was a patron of literature and the arts. Ain-e Akbari and Akbarnama written by his
court historian Abul Fazl are the best examples of literature of the Medieval period.
Therefore, option (c) is the correct answer.

Q95.
Answer: d
Explanation:
● The sites of Ukshi, Jambharun, Kashel, and Barsu are known for prehistoric geoglyphs.
Geoglyphs are a form of prehistoric rock art, created on the surface of laterite plateaus.
They are made by either positioning rocks, or rock fragments, or by reduction technique
i.e., removing a part of the rock surface through an incision, picking, carving, or abrading.
● They can be in the form of rock paintings, etchings, cup marks, and ring marks. The
figures depicted in the geoglyphs include humans and animals. These are the evidence
pointing to the presence of human settlement in the Konkan region from the Mesolithic
era till the Early Historic era and possibly, were contemporary to other Deccan Chalcolithic
cultures.
● Ratnagiri district (Konkan region) has numerous prehistoric geoglyphs, also called “Katal
Shilpa. The largest geoglyph in India is at Kasheli which has a large figure of an elephant.
● UNESCO’s tentative world heritage list mentions seven sites with geoglyphs in the
Konkan region of Ratnagiri district — Ukshi, Jambharun, Kasheli, Rundhe Tali, Devihsol,
Barsu, and Devache Gothane.
Therefore, option (d) is the correct answer.
Relevance: Experts have raised concerns over the proposed location for the mega oil refinery in
Barsu village of Maharashtra citing damage to prehistoric geoglyphs found in the area.

Q96.
Answer: c
Explanation:
● The work on the walls or a solid structure are referred to as Murals. These have existed
in India since ancient times and can be dated between 10th century BC and 10th century
AD.
● Ravan Chhaya Rock Shelter is located in the Keonjhar district of Odisha. The ancient
mural paintings on a rock shelter are in the shape of a half-opened umbrella. It is believed
that this shelter acted like a royal hunting lodge. The most noticeable painting is that of
a royal procession that dates back to the 7th century AD. So, point 1 is correct.
● Lepakshi Temple paintings are located in the Anantapur district of Andhra Pradesh.
These mural paintings were executed on Veerabhadra temple walls at Lepakshi in the
16th century. Made during the Vijayanagara period, they follow a religious theme based
_
on Ramayana, Mahabharata and incarnations of Vishnu. The paintings show a complete
absence of primary colours, especially blue. So, point 2 is correct.
● Ajanta Cave Paintings is one of the oldest surviving murals of the Indian subcontinent.
Ajanta caves were carved between the 2nd century BC and 5th century AD out of volcanic
rocks. It consists of a set of 29 caves, carved in a horseshoe shape. These are Buddhist
caves with exquisite mural paintings. The paintings portray human values and social
fabric along with styles, costumes and ornaments of that period. The common themes of
these paintings range from Jataka stories to the life of Buddha to elaborate decorative
patterns of flora and fauna. So, point 3 is correct.
● The Khajuraho Group of Monuments are a group of Hindu and Jain temples in Madhya
Pradesh. They are recognised as UNESCO World Heritage Sites. The temples are famous
for their nagara-style architectural symbolism and a few erotic sculptures. There is no
record of mural paintings in the Khajuraho Group of Monuments. So, point 4 is not
correct.
Therefore, option (c) is the correct answer.

Q97.
Answer: d
Explanation:
● There are two breeds of pashmina-yielding goats viz. Chegu and Changthangi in India.
Both of these breeds are found at high altitudes in the trans-Himalayan Mountain range
bordering Tibet.
● Changthangi goats are localised in the Changthang sub-division of the Ladakh region of
Jammu and Kashmir.
● It is also known as the Changra goat. The breed is raised mainly for its ultra-fine wool
production. The breed was also reared for meat production in the past. Changthangi
goats grow thick, warm undercoats.
● It is usually domesticated and raised by nomadic communities called the Changpa in the
Changthang region of Greater Ladakh, Kashmir. It has been listed on ‘endangered species
list’ by the National Bureau of Animal Genetic Resources.
● Pashmina Shawls are hand woven by skilled Artisans of Kashmir. The weaving requires a
lot of expertise and experienced hands for creating find such fine embroidery. This art of
weaving Pashmina Shawls is passed on from generation to generation as a tradition in
Kashmir.
Therefore, option (d) is the correct answer.
Relevance: Custom officials complained about the presence of ‘Shahtoosh’ guard hair in the
Pashmina shawl, which is obtained from endangered Tibetan antelopes in many of their export
consignments.

Q98.
Answer: d
Explanation:
● Archaeological Survey of India: It was founded in 1861 by the British colonial
administration, Alexander Cunningham. After independence, it came under the Ancient
_
Monuments and Archaeological Sites and Remains Act of 1958. So, pair 1 is correctly
matched.
● Asiatic Society of Bengal: It was established in 1784 at the initiative of its Founder
President William Jones. The name of the society underwent several changes such as the
Asiatic Society of Bengal (1832-1935), the Royal Asiatic Society of Bengal (1936-1951) and
in July 1952 it came to be known as the Asiatic Society. Since 1984 the Asiatic Society has
been declared an Institution of National Importance by an Act of the Parliament of India.
So, pair 2 is correctly matched.
● Calcutta Medical College: In 1833 William Bentinck recommended a requirement of a
medical college 'for the education of the natives'. Consequently, a new medical college,
known as the Calcutta Medical College was established in 1835. Its purpose was to train
native youths irrespective of caste and creed. So, pair 3 is correctly matched.
So, all three pairs are correctly matched.
Therefore, option (d) is the correct answer.

Q99.
Answer: c
Explanation:
● In order to counter the threat of working-class people's participation in the anti-Simon
agitation towards the end of the 1920s, the British government appointed the Royal
Commission on Labour (Whitley Commission on Labour) in 1929 to visit India and suggest
measures for improving labour relations and promoting labour welfare.
● The committee was to enquire into and to make recommendations on the conditions of
health, efficiency and standard of living of the workers and on the relations between
employers and employed in industrial undertakings and plantations in British India.
● Many labour welfare legislations and industrial laws like the Trade Union Act 1926,
Industrial Dispute Act 1947, the Payment of Wages Act 1936, Minimum Wages Act 1948,
etc. owe the origin in some way or the other to this report.
Therefore, option (c) is the correct answer.

Q100.
Answer: b
Explanation:
Fog can form in the following situations:
● Radiation fog: It is most prevalent during the fall and winter. It forms overnight as the air
near the ground cools and stabilizes. When this cooling causes the air to reach saturation,
fog will form. So, point 1 is correct.
● Convection fog: This type of fog occurs when warm, moist air is cooled as it rises, often
due to the presence of a mountain or other topographic feature. This can also occur when
the air is cooled by contact with a cold surface, such as a cold ocean or lake. So, point 2
is correct.
● Advection fog: This type of fog occurs when moist air is blown over a cold surface, such
as when warm, moist air from the ocean blows over a colder land mass. So, point 3 is not
correct.
_
● Upslope fog: This type of fog occurs when moist air is forced to rise up a slope, such as a
mountain, and cools as it rises, causing the moisture in the air to condense and form fog.
● Steam fog: This type of fog occurs when very cold air moves over a warmer body of water,
causing the moisture in the warmer air to condense and form fog.
Therefore, option (b) is the correct answer.

You might also like